Quiz-summary
0 of 30 questions completed
Questions:
- 1
- 2
- 3
- 4
- 5
- 6
- 7
- 8
- 9
- 10
- 11
- 12
- 13
- 14
- 15
- 16
- 17
- 18
- 19
- 20
- 21
- 22
- 23
- 24
- 25
- 26
- 27
- 28
- 29
- 30
Information
Premium Practice Questions
You have already completed the quiz before. Hence you can not start it again.
Quiz is loading...
You must sign in or sign up to start the quiz.
You have to finish following quiz, to start this quiz:
Results
0 of 30 questions answered correctly
Your time:
Time has elapsed
Categories
- Not categorized 0%
- 1
- 2
- 3
- 4
- 5
- 6
- 7
- 8
- 9
- 10
- 11
- 12
- 13
- 14
- 15
- 16
- 17
- 18
- 19
- 20
- 21
- 22
- 23
- 24
- 25
- 26
- 27
- 28
- 29
- 30
- Answered
- Review
-
Question 1 of 30
1. Question
A patient is admitted to the hospital with complaints of vomiting blood and passing dark, tarry stools. The physician’s initial assessment notes “hematemesis” and “melena.” Considering the principles of accurate medical coding as taught at CMCB University, which ICD-10-CM code best represents the documented clinical presentation if the precise source of the gastrointestinal bleeding is not yet identified?
Correct
The scenario describes a patient presenting with symptoms suggestive of a gastrointestinal bleed. The physician documents “hematemesis” and “melena.” Hematemesis refers to the vomiting of blood, and melena refers to the passage of dark, tarry stools, which indicates digested blood. In ICD-10-CM, the primary diagnosis for hematemesis is K92.0, and for melena, it is K92.1. When both are present and indicative of a bleeding gastrointestinal tract, the coding guidelines direct us to code the underlying cause if known. However, if the specific site of bleeding is not identified, but the symptoms are clearly documented, coding for the symptoms themselves is appropriate. The question asks for the most accurate ICD-10-CM code for the documented findings. While K92.0 and K92.1 are specific to the symptoms, the overarching condition of gastrointestinal hemorrhage, unspecified, is represented by K92.2. This code is used when the documentation indicates bleeding from the GI tract but does not specify the exact location or cause, which aligns with the presented clinical picture where only the symptoms are explicitly stated without further diagnostic detail. Therefore, K92.2 is the most encompassing and appropriate code in this context for a medical coder at CMCB University to assign, reflecting the principle of coding to the highest specificity supported by documentation.
Incorrect
The scenario describes a patient presenting with symptoms suggestive of a gastrointestinal bleed. The physician documents “hematemesis” and “melena.” Hematemesis refers to the vomiting of blood, and melena refers to the passage of dark, tarry stools, which indicates digested blood. In ICD-10-CM, the primary diagnosis for hematemesis is K92.0, and for melena, it is K92.1. When both are present and indicative of a bleeding gastrointestinal tract, the coding guidelines direct us to code the underlying cause if known. However, if the specific site of bleeding is not identified, but the symptoms are clearly documented, coding for the symptoms themselves is appropriate. The question asks for the most accurate ICD-10-CM code for the documented findings. While K92.0 and K92.1 are specific to the symptoms, the overarching condition of gastrointestinal hemorrhage, unspecified, is represented by K92.2. This code is used when the documentation indicates bleeding from the GI tract but does not specify the exact location or cause, which aligns with the presented clinical picture where only the symptoms are explicitly stated without further diagnostic detail. Therefore, K92.2 is the most encompassing and appropriate code in this context for a medical coder at CMCB University to assign, reflecting the principle of coding to the highest specificity supported by documentation.
-
Question 2 of 30
2. Question
A patient visits the CMCB University Health Clinic complaining of painful urination and an increased urge to urinate. The physician’s progress note meticulously records these as “dysuria” and “frequency.” No definitive diagnosis of a specific urinary tract infection or other condition has been made at this initial encounter. Which of the following ICD-10-CM code combinations most accurately represents the physician’s documented findings for this patient’s visit?
Correct
The scenario describes a patient presenting with symptoms indicative of a urinary tract infection (UTI). The physician documents “dysuria” and “frequency” as the primary complaints. In ICD-10-CM, dysuria is coded to R30.0, and urinary frequency is coded to R35.0. According to ICD-10-CM Official Guidelines for Coding and Reporting, when a patient presents with signs and symptoms that are integral to a diagnosed condition, the diagnosed condition should be coded. However, if the signs and symptoms are the only information available and a definitive diagnosis has not been established, the signs and symptoms themselves are coded. In this case, the physician has not provided a definitive diagnosis for the UTI, only the presenting symptoms. Therefore, the appropriate coding approach is to report the symptoms. The question asks for the most accurate coding for the physician’s documentation. The combination of R30.0 and R35.0 accurately reflects the documented symptoms. The explanation of why this is the correct approach involves understanding the hierarchy of coding: definitive diagnoses take precedence over signs and symptoms. However, in the absence of a definitive diagnosis, coding the documented symptoms is the standard practice. This aligns with the principles of accurate and complete medical coding taught at CMCB University, emphasizing the importance of reflecting the physician’s documentation precisely. The ability to discern when to code symptoms versus diagnoses is a critical skill for medical coders, directly impacting reimbursement and patient record accuracy. This question tests the understanding of these fundamental coding principles within the context of common medical presentations.
Incorrect
The scenario describes a patient presenting with symptoms indicative of a urinary tract infection (UTI). The physician documents “dysuria” and “frequency” as the primary complaints. In ICD-10-CM, dysuria is coded to R30.0, and urinary frequency is coded to R35.0. According to ICD-10-CM Official Guidelines for Coding and Reporting, when a patient presents with signs and symptoms that are integral to a diagnosed condition, the diagnosed condition should be coded. However, if the signs and symptoms are the only information available and a definitive diagnosis has not been established, the signs and symptoms themselves are coded. In this case, the physician has not provided a definitive diagnosis for the UTI, only the presenting symptoms. Therefore, the appropriate coding approach is to report the symptoms. The question asks for the most accurate coding for the physician’s documentation. The combination of R30.0 and R35.0 accurately reflects the documented symptoms. The explanation of why this is the correct approach involves understanding the hierarchy of coding: definitive diagnoses take precedence over signs and symptoms. However, in the absence of a definitive diagnosis, coding the documented symptoms is the standard practice. This aligns with the principles of accurate and complete medical coding taught at CMCB University, emphasizing the importance of reflecting the physician’s documentation precisely. The ability to discern when to code symptoms versus diagnoses is a critical skill for medical coders, directly impacting reimbursement and patient record accuracy. This question tests the understanding of these fundamental coding principles within the context of common medical presentations.
-
Question 3 of 30
3. Question
A patient visits the CMCB University Health Clinic complaining of dysuria, increased urinary frequency, and suprapubic pain. The physician orders a urinalysis, which shows positive leukocytes and nitrites. A subsequent urine culture and sensitivity test identifies *Escherichia coli* and indicates susceptibility to nitrofurantoin. Based on this clinical information and the principles of ICD-10-CM coding as taught at CMCB University, which code most accurately represents the patient’s primary diagnosis?
Correct
The scenario describes a patient presenting with symptoms indicative of a urinary tract infection (UTI). The physician performs a urinalysis and a urine culture and sensitivity (C&S) test. The urinalysis reveals leukocytes and nitrites, which are common indicators of a UTI. The urine culture identifies *Escherichia coli* (E. coli) as the causative agent and the sensitivity testing shows that E. coli is susceptible to nitrofurantoin. In ICD-10-CM coding, the primary diagnosis for a UTI is typically coded using a category within N39.0 (Urinary tract infection, site not specified). However, when a specific organism is identified and documented, and the site is also specified (even if generally as “urinary tract”), more specific coding might be applicable if the documentation supports it. Given the identification of E. coli, a common pathogen for UTIs, and the sensitivity to nitrofurantoin, the coder must select the most accurate ICD-10-CM code. The question tests the understanding of how to code for a UTI when the causative organism and its sensitivity are documented. While N39.0 is a general code, the presence of specific microbiological findings can sometimes lead to more precise coding if the documentation allows. However, without a more specific site documented (e.g., cystitis, pyelonephritis), N39.0 remains the most appropriate primary code for a general UTI. The sensitivity report for nitrofurantoin is crucial for the physician’s treatment plan but does not directly alter the ICD-10-CM diagnosis code for the infection itself. The focus for diagnosis coding is on the condition and its manifestations as documented. Therefore, the most accurate ICD-10-CM code reflecting a confirmed UTI with identified organism and sensitivity, but without a more specific anatomical site beyond “urinary tract,” is N39.0.
Incorrect
The scenario describes a patient presenting with symptoms indicative of a urinary tract infection (UTI). The physician performs a urinalysis and a urine culture and sensitivity (C&S) test. The urinalysis reveals leukocytes and nitrites, which are common indicators of a UTI. The urine culture identifies *Escherichia coli* (E. coli) as the causative agent and the sensitivity testing shows that E. coli is susceptible to nitrofurantoin. In ICD-10-CM coding, the primary diagnosis for a UTI is typically coded using a category within N39.0 (Urinary tract infection, site not specified). However, when a specific organism is identified and documented, and the site is also specified (even if generally as “urinary tract”), more specific coding might be applicable if the documentation supports it. Given the identification of E. coli, a common pathogen for UTIs, and the sensitivity to nitrofurantoin, the coder must select the most accurate ICD-10-CM code. The question tests the understanding of how to code for a UTI when the causative organism and its sensitivity are documented. While N39.0 is a general code, the presence of specific microbiological findings can sometimes lead to more precise coding if the documentation allows. However, without a more specific site documented (e.g., cystitis, pyelonephritis), N39.0 remains the most appropriate primary code for a general UTI. The sensitivity report for nitrofurantoin is crucial for the physician’s treatment plan but does not directly alter the ICD-10-CM diagnosis code for the infection itself. The focus for diagnosis coding is on the condition and its manifestations as documented. Therefore, the most accurate ICD-10-CM code reflecting a confirmed UTI with identified organism and sensitivity, but without a more specific anatomical site beyond “urinary tract,” is N39.0.
-
Question 4 of 30
4. Question
During a routine outpatient visit at CMCB University Medical Center, a patient, Mr. Alistair Finch, presented with complaints of intermittent rectal bleeding and dark, tarry stools over the past week. Following a thorough history and physical examination, the physician ordered a diagnostic colonoscopy to investigate the source of the bleeding. During the colonoscopy, a single 1.5 cm sessile polyp was identified in the sigmoid colon and was successfully removed using a snare technique. The pathology report for the polyp is pending. Considering the principles of accurate medical coding and billing as taught at CMCB University, which combination of ICD-10-CM and CPT codes best represents this encounter for initial billing purposes, assuming no other significant findings or procedures were documented?
Correct
The scenario describes a patient presenting with symptoms indicative of a gastrointestinal bleed. The physician performs a diagnostic colonoscopy to identify the source of bleeding and also excises a polyp found during the procedure. For coding purposes, we need to identify the primary reason for the encounter and the procedures performed. The patient’s initial presentation is with hematochezia, which is a symptom. The underlying cause of the hematochezia is not definitively known at the time of the initial encounter, but the colonoscopy is performed to investigate this symptom. Therefore, the symptom of hematochezia, classified under ICD-10-CM code K92.2 (Gastrointestinal hemorrhage, unspecified), is the primary reason for the encounter. The physician performed a colonoscopy with a biopsy and polypectomy. According to CPT coding guidelines, when a colonoscopy is performed and a polyp is removed, the appropriate code for the colonoscopy with polypectomy should be reported. The specific CPT code for a colonoscopy with removal of a polyp by snare technique is 45385. The documentation indicates a polyp was found and removed, and the method of removal (snare) is implied by the standard practice for such procedures. Therefore, the correct coding combination involves reporting the ICD-10-CM code for the symptom that necessitated the diagnostic procedure and the CPT code for the diagnostic procedure with the therapeutic intervention. The ICD-10-CM code K92.2 accurately reflects the unspecified gastrointestinal hemorrhage leading to the colonoscopy. The CPT code 45385 accurately captures the colonoscopy with polypectomy.
Incorrect
The scenario describes a patient presenting with symptoms indicative of a gastrointestinal bleed. The physician performs a diagnostic colonoscopy to identify the source of bleeding and also excises a polyp found during the procedure. For coding purposes, we need to identify the primary reason for the encounter and the procedures performed. The patient’s initial presentation is with hematochezia, which is a symptom. The underlying cause of the hematochezia is not definitively known at the time of the initial encounter, but the colonoscopy is performed to investigate this symptom. Therefore, the symptom of hematochezia, classified under ICD-10-CM code K92.2 (Gastrointestinal hemorrhage, unspecified), is the primary reason for the encounter. The physician performed a colonoscopy with a biopsy and polypectomy. According to CPT coding guidelines, when a colonoscopy is performed and a polyp is removed, the appropriate code for the colonoscopy with polypectomy should be reported. The specific CPT code for a colonoscopy with removal of a polyp by snare technique is 45385. The documentation indicates a polyp was found and removed, and the method of removal (snare) is implied by the standard practice for such procedures. Therefore, the correct coding combination involves reporting the ICD-10-CM code for the symptom that necessitated the diagnostic procedure and the CPT code for the diagnostic procedure with the therapeutic intervention. The ICD-10-CM code K92.2 accurately reflects the unspecified gastrointestinal hemorrhage leading to the colonoscopy. The CPT code 45385 accurately captures the colonoscopy with polypectomy.
-
Question 5 of 30
5. Question
A patient visits the CMCB University Health Clinic complaining of painful urination and a persistent urge to urinate frequently. The physician’s progress note explicitly states “dysuria” and “urinary frequency” as the presenting symptoms, but no definitive diagnosis of a urinary tract infection has been made at this visit. Which combination of ICD-10-CM codes accurately reflects the documented clinical information for this encounter, adhering to the principles of accurate medical coding as taught at CMCB University?
Correct
The scenario describes a patient presenting with symptoms indicative of a urinary tract infection (UTI). The physician’s documentation notes “dysuria” and “frequency.” In ICD-10-CM, dysuria is coded as R30.0, and urinary frequency is coded as R35.0. According to ICD-10-CM Official Guidelines for Coding and Reporting, when a patient has signs and symptoms that are integral to a diagnosed condition, and the signs and symptoms are not otherwise specified, the diagnosed condition should be coded. However, in this case, the physician has documented specific symptoms (dysuria and frequency) without a definitive diagnosis of a UTI. The guidelines also state that signs and symptoms that are not routinely associated with a disease process should be coded when they are present. Dysuria and frequency are common symptoms of a UTI, but they can also be indicative of other conditions. Since the physician has not explicitly diagnosed a UTI, and these symptoms are documented as the presenting complaints, the appropriate coding approach is to report the symptoms individually. Therefore, the correct coding would involve R30.0 for dysuria and R35.0 for urinary frequency. The question tests the understanding of coding guidelines when signs and symptoms are documented but a definitive diagnosis is not yet established, emphasizing the importance of accurate representation of the patient’s condition based on the provided documentation. This aligns with the rigorous standards of accurate coding expected at CMCB University, where understanding the nuances of ICD-10-CM is paramount for proper reimbursement and patient care tracking. The ability to differentiate between coding a symptom versus a confirmed diagnosis is a foundational skill for any medical coder.
Incorrect
The scenario describes a patient presenting with symptoms indicative of a urinary tract infection (UTI). The physician’s documentation notes “dysuria” and “frequency.” In ICD-10-CM, dysuria is coded as R30.0, and urinary frequency is coded as R35.0. According to ICD-10-CM Official Guidelines for Coding and Reporting, when a patient has signs and symptoms that are integral to a diagnosed condition, and the signs and symptoms are not otherwise specified, the diagnosed condition should be coded. However, in this case, the physician has documented specific symptoms (dysuria and frequency) without a definitive diagnosis of a UTI. The guidelines also state that signs and symptoms that are not routinely associated with a disease process should be coded when they are present. Dysuria and frequency are common symptoms of a UTI, but they can also be indicative of other conditions. Since the physician has not explicitly diagnosed a UTI, and these symptoms are documented as the presenting complaints, the appropriate coding approach is to report the symptoms individually. Therefore, the correct coding would involve R30.0 for dysuria and R35.0 for urinary frequency. The question tests the understanding of coding guidelines when signs and symptoms are documented but a definitive diagnosis is not yet established, emphasizing the importance of accurate representation of the patient’s condition based on the provided documentation. This aligns with the rigorous standards of accurate coding expected at CMCB University, where understanding the nuances of ICD-10-CM is paramount for proper reimbursement and patient care tracking. The ability to differentiate between coding a symptom versus a confirmed diagnosis is a foundational skill for any medical coder.
-
Question 6 of 30
6. Question
A patient visits CMCB University’s affiliated clinic complaining of persistent upper abdominal discomfort. The physician’s notes detail the patient’s subjective experience of “gastric distress” and document the performance of an esophagogastroduodenoscopy (EGD) with the collection of tissue samples for pathological examination. Which CPT code most accurately reflects the physician’s documented procedure, considering the diagnostic intent and the interventional component?
Correct
The scenario describes a patient presenting with symptoms indicative of a gastrointestinal issue. The physician’s documentation notes “gastric distress” and orders an esophagogastroduodenoscopy (EGD) with biopsy. For the EGD, the primary CPT code is 43239, which represents “Esophagogastroduodenoscopy, flexible, with biopsy, single or multiple.” The documentation explicitly states a biopsy was performed. The diagnosis documented is “gastric distress,” which is a symptom. In ICD-10-CM, symptoms are coded when a definitive diagnosis is not established. While “gastric distress” could be mapped to R10.13 (Epigastric pain), the more appropriate and specific code for generalized abdominal pain, which often encompasses gastric distress when a more specific diagnosis isn’t provided, is R10.84 (Generalized abdominal pain). However, the question focuses on the procedural coding and the implication of the biopsy. The key is to identify the most comprehensive code for the procedure performed. The EGD itself is coded, and the addition of a biopsy is a crucial detail that modifies the base code. Therefore, 43239 accurately reflects the performed service. The rationale for selecting this code over others lies in its specificity for an EGD that includes a biopsy. Other codes might represent an EGD without biopsy (43235), or a different endoscopic procedure entirely. The diagnosis code, while important for billing, does not alter the procedural code itself. The question tests the ability to link documented procedures and findings to the correct CPT codes, understanding that the presence of a biopsy necessitates a specific code that includes this service. This aligns with the rigorous standards of accurate coding expected at CMCB University, where precise application of coding guidelines is paramount for reimbursement and patient care integrity.
Incorrect
The scenario describes a patient presenting with symptoms indicative of a gastrointestinal issue. The physician’s documentation notes “gastric distress” and orders an esophagogastroduodenoscopy (EGD) with biopsy. For the EGD, the primary CPT code is 43239, which represents “Esophagogastroduodenoscopy, flexible, with biopsy, single or multiple.” The documentation explicitly states a biopsy was performed. The diagnosis documented is “gastric distress,” which is a symptom. In ICD-10-CM, symptoms are coded when a definitive diagnosis is not established. While “gastric distress” could be mapped to R10.13 (Epigastric pain), the more appropriate and specific code for generalized abdominal pain, which often encompasses gastric distress when a more specific diagnosis isn’t provided, is R10.84 (Generalized abdominal pain). However, the question focuses on the procedural coding and the implication of the biopsy. The key is to identify the most comprehensive code for the procedure performed. The EGD itself is coded, and the addition of a biopsy is a crucial detail that modifies the base code. Therefore, 43239 accurately reflects the performed service. The rationale for selecting this code over others lies in its specificity for an EGD that includes a biopsy. Other codes might represent an EGD without biopsy (43235), or a different endoscopic procedure entirely. The diagnosis code, while important for billing, does not alter the procedural code itself. The question tests the ability to link documented procedures and findings to the correct CPT codes, understanding that the presence of a biopsy necessitates a specific code that includes this service. This aligns with the rigorous standards of accurate coding expected at CMCB University, where precise application of coding guidelines is paramount for reimbursement and patient care integrity.
-
Question 7 of 30
7. Question
A patient is seen at CMCB University’s teaching hospital for persistent upper abdominal discomfort. The physician’s progress note states the diagnosis as “gastritis.” The note does not specify if the gastritis is acute or chronic, nor does it mention any associated complications such as bleeding or obstruction. Which ICD-10-CM code most accurately reflects the documented diagnosis for billing and record-keeping purposes within the CMCB University healthcare system?
Correct
The scenario describes a patient presenting with symptoms indicative of a gastrointestinal issue. The physician documents a diagnosis of “gastritis,” which is an inflammation of the stomach lining. To accurately code this condition for billing purposes at CMCB University’s affiliated clinics, the medical coder must consult the ICD-10-CM coding system. Gastritis is a condition that falls under diseases of the digestive system. Specifically, ICD-10-CM categorizes gastritis under K29.0, which is “Acute gastritis.” However, the documentation does not specify whether the gastritis is acute or chronic. In such cases, the coding guidelines direct the coder to use the most specific code available based on the provided documentation. If the physician had documented “chronic gastritis,” the code would be K29.5. If the physician had documented “gastritis with bleeding,” the code would be K29.01 for acute or K29.51 for chronic. Since only “gastritis” is documented without further specification of acuity or associated conditions like bleeding or obstruction, the most appropriate and specific code reflecting unspecified gastritis is K29.3, “Other and unspecified gastritis.” This code accurately represents the physician’s documented diagnosis without adding information not present in the medical record, adhering to the principle of coding to the highest level of specificity supported by documentation. This ensures accurate reimbursement and compliance with coding standards taught at CMCB University.
Incorrect
The scenario describes a patient presenting with symptoms indicative of a gastrointestinal issue. The physician documents a diagnosis of “gastritis,” which is an inflammation of the stomach lining. To accurately code this condition for billing purposes at CMCB University’s affiliated clinics, the medical coder must consult the ICD-10-CM coding system. Gastritis is a condition that falls under diseases of the digestive system. Specifically, ICD-10-CM categorizes gastritis under K29.0, which is “Acute gastritis.” However, the documentation does not specify whether the gastritis is acute or chronic. In such cases, the coding guidelines direct the coder to use the most specific code available based on the provided documentation. If the physician had documented “chronic gastritis,” the code would be K29.5. If the physician had documented “gastritis with bleeding,” the code would be K29.01 for acute or K29.51 for chronic. Since only “gastritis” is documented without further specification of acuity or associated conditions like bleeding or obstruction, the most appropriate and specific code reflecting unspecified gastritis is K29.3, “Other and unspecified gastritis.” This code accurately represents the physician’s documented diagnosis without adding information not present in the medical record, adhering to the principle of coding to the highest level of specificity supported by documentation. This ensures accurate reimbursement and compliance with coding standards taught at CMCB University.
-
Question 8 of 30
8. Question
A patient at CMCB University Medical Center presents with severe epigastric pain and hematemesis. A diagnostic upper gastrointestinal endoscopy is performed, revealing a bleeding duodenal ulcer. The physician successfully cauterizes the bleeding site. Which combination of ICD-10-CM and CPT codes best represents this encounter for billing and record-keeping purposes at CMCB University?
Correct
The scenario describes a patient presenting with symptoms indicative of a gastrointestinal bleed, specifically hematemesis (vomiting blood). The physician performs a diagnostic upper endoscopy to visualize the upper digestive tract and identify the source of bleeding. During the procedure, a bleeding duodenal ulcer is found and treated with cauterization. For coding purposes, the primary diagnosis is the duodenal ulcer. According to ICD-10-CM guidelines, when a definitive diagnosis is made during a diagnostic procedure, that diagnosis should be coded. Duodenal ulcers are classified under category K26. The specific type of ulcer, if documented, would further refine the code. Assuming the documentation specifies a non-specified duodenal ulcer without hemorrhage or perforation, the appropriate ICD-10-CM code would be K26.9 (Duodenal ulcer, unspecified as acute or chronic, without hemorrhage or perforation). The procedure performed is an upper gastrointestinal endoscopy with cauterization. CPT codes are used to report procedures. An upper endoscopy is reported with codes from the 43235-43259 range. Code 43239 specifically reports esophagogastroduodenoscopy (EGD) with endoscopic ultrasound examination, which is not described. Code 43247 reports EGD with injection, ligation, or application of a hot or cold metallic probe or clips, which aligns with cauterization. Therefore, 43247 is the most appropriate CPT code for the procedure. The question asks for the most accurate coding combination for the physician’s encounter. This involves selecting the correct ICD-10-CM code for the diagnosis and the correct CPT code for the procedure. The combination of K26.9 for the duodenal ulcer and 43247 for the EGD with cauterization accurately reflects the clinical encounter and adheres to coding principles taught at CMCB University, emphasizing the importance of linking diagnoses to procedures for proper reimbursement and medical necessity.
Incorrect
The scenario describes a patient presenting with symptoms indicative of a gastrointestinal bleed, specifically hematemesis (vomiting blood). The physician performs a diagnostic upper endoscopy to visualize the upper digestive tract and identify the source of bleeding. During the procedure, a bleeding duodenal ulcer is found and treated with cauterization. For coding purposes, the primary diagnosis is the duodenal ulcer. According to ICD-10-CM guidelines, when a definitive diagnosis is made during a diagnostic procedure, that diagnosis should be coded. Duodenal ulcers are classified under category K26. The specific type of ulcer, if documented, would further refine the code. Assuming the documentation specifies a non-specified duodenal ulcer without hemorrhage or perforation, the appropriate ICD-10-CM code would be K26.9 (Duodenal ulcer, unspecified as acute or chronic, without hemorrhage or perforation). The procedure performed is an upper gastrointestinal endoscopy with cauterization. CPT codes are used to report procedures. An upper endoscopy is reported with codes from the 43235-43259 range. Code 43239 specifically reports esophagogastroduodenoscopy (EGD) with endoscopic ultrasound examination, which is not described. Code 43247 reports EGD with injection, ligation, or application of a hot or cold metallic probe or clips, which aligns with cauterization. Therefore, 43247 is the most appropriate CPT code for the procedure. The question asks for the most accurate coding combination for the physician’s encounter. This involves selecting the correct ICD-10-CM code for the diagnosis and the correct CPT code for the procedure. The combination of K26.9 for the duodenal ulcer and 43247 for the EGD with cauterization accurately reflects the clinical encounter and adheres to coding principles taught at CMCB University, emphasizing the importance of linking diagnoses to procedures for proper reimbursement and medical necessity.
-
Question 9 of 30
9. Question
A patient presents to the emergency department with complaints of dark, tarry stools and visible blood in their stool. The physician’s initial assessment notes “melena” and “hematochezia” and orders an esophagogastroduodenoscopy (EGD) to investigate the source of the bleeding. Considering the documentation and the planned diagnostic procedure, what is the most appropriate ICD-10-CM code to represent the patient’s primary diagnosis at this stage of evaluation for CMCB University’s advanced coding curriculum?
Correct
The scenario describes a patient presenting with symptoms suggestive of a gastrointestinal bleed. The physician documents “melena” and “hematochezia.” Melena refers to dark, tarry stools, typically indicative of upper gastrointestinal bleeding where blood has been digested. Hematochezia refers to bright red blood in the stool, usually indicating lower gastrointestinal bleeding. The physician also orders an esophagogastroduodenoscopy (EGD) to visualize the upper digestive tract and identify the source of bleeding. The EGD procedure is coded using CPT code 43239, which represents an esophagogastroduodenoscopy with biopsy, submucosal injection, lesion removal by snare, or cauterization. The diagnosis of a gastrointestinal bleed, unspecified site, is coded using ICD-10-CM code K92.2. The question asks to identify the most appropriate ICD-10-CM code for the *diagnosis* based on the provided documentation and the physician’s diagnostic procedure. While the physician is investigating a gastrointestinal bleed, the documentation specifically mentions “melena” and “hematochezia,” which are signs of bleeding. The EGD is a procedure to diagnose the *cause* of the bleeding. The ICD-10-CM code K92.2, “Gastrointestinal hemorrhage, unspecified,” is the most accurate code to represent the patient’s condition as documented, given that the specific site or cause of the bleeding has not yet been definitively identified by the physician’s documentation at the time of this encounter, even though an EGD is planned. The other options represent more specific diagnoses or symptoms that are not directly stated as the primary diagnosis in the provided context. For instance, K62.5 refers to hemorrhage of the anus and rectum, which is a specific location not confirmed by the documentation. K29.00 refers to acute gastritis without bleeding, which is incorrect as bleeding is present. K92.0 refers to hematemesis, which is vomiting blood, a symptom not explicitly stated as the primary complaint. Therefore, K92.2 accurately reflects the unspecified nature of the gastrointestinal hemorrhage as presented.
Incorrect
The scenario describes a patient presenting with symptoms suggestive of a gastrointestinal bleed. The physician documents “melena” and “hematochezia.” Melena refers to dark, tarry stools, typically indicative of upper gastrointestinal bleeding where blood has been digested. Hematochezia refers to bright red blood in the stool, usually indicating lower gastrointestinal bleeding. The physician also orders an esophagogastroduodenoscopy (EGD) to visualize the upper digestive tract and identify the source of bleeding. The EGD procedure is coded using CPT code 43239, which represents an esophagogastroduodenoscopy with biopsy, submucosal injection, lesion removal by snare, or cauterization. The diagnosis of a gastrointestinal bleed, unspecified site, is coded using ICD-10-CM code K92.2. The question asks to identify the most appropriate ICD-10-CM code for the *diagnosis* based on the provided documentation and the physician’s diagnostic procedure. While the physician is investigating a gastrointestinal bleed, the documentation specifically mentions “melena” and “hematochezia,” which are signs of bleeding. The EGD is a procedure to diagnose the *cause* of the bleeding. The ICD-10-CM code K92.2, “Gastrointestinal hemorrhage, unspecified,” is the most accurate code to represent the patient’s condition as documented, given that the specific site or cause of the bleeding has not yet been definitively identified by the physician’s documentation at the time of this encounter, even though an EGD is planned. The other options represent more specific diagnoses or symptoms that are not directly stated as the primary diagnosis in the provided context. For instance, K62.5 refers to hemorrhage of the anus and rectum, which is a specific location not confirmed by the documentation. K29.00 refers to acute gastritis without bleeding, which is incorrect as bleeding is present. K92.0 refers to hematemesis, which is vomiting blood, a symptom not explicitly stated as the primary complaint. Therefore, K92.2 accurately reflects the unspecified nature of the gastrointestinal hemorrhage as presented.
-
Question 10 of 30
10. Question
A patient visits the CMCB University Health Clinic complaining of painful urination, increased frequency of urination, and a persistent urge to urinate. The physician’s progress note documents these symptoms and also notes the absence of blood in the urine. A urinalysis with microscopy was performed. Which ICD-10-CM code best represents the patient’s primary diagnosed condition as documented in the encounter notes?
Correct
The scenario describes a patient presenting with symptoms indicative of a urinary tract infection (UTI), specifically cystitis. The physician’s documentation notes dysuria, frequency, and urgency. For coding purposes, the primary diagnosis is the symptomatic presentation of the UTI. ICD-10-CM code N30.00 is assigned for cystitis without hematuria. The physician also performed a urinalysis with microscopy, which is a diagnostic procedure. The CPT code for a urinalysis with microscopy is 81001. However, the question asks for the *most appropriate* code for the *patient’s condition* as documented, not the procedure performed. Therefore, the focus should be on the diagnosis. The documentation clearly points to cystitis. While other conditions might be considered in a differential diagnosis, the documented symptoms directly support cystitis. The absence of hematuria is also noted, which is crucial for selecting the correct subcategory within N30.0-. Therefore, N30.00 accurately reflects the documented condition. The explanation of why this is the correct choice involves understanding the ICD-10-CM coding conventions for genitourinary system diseases and the importance of precise documentation to select the most specific code. A thorough understanding of medical terminology, particularly terms related to the urinary system and inflammation, is essential. Furthermore, recognizing the distinction between a diagnosis code and a procedure code is fundamental for accurate medical billing and coding, a core competency at CMCB University. The correct approach involves analyzing the physician’s notes to identify the primary condition being treated and then locating the most specific ICD-10-CM code that reflects that condition, considering any qualifying details provided in the documentation.
Incorrect
The scenario describes a patient presenting with symptoms indicative of a urinary tract infection (UTI), specifically cystitis. The physician’s documentation notes dysuria, frequency, and urgency. For coding purposes, the primary diagnosis is the symptomatic presentation of the UTI. ICD-10-CM code N30.00 is assigned for cystitis without hematuria. The physician also performed a urinalysis with microscopy, which is a diagnostic procedure. The CPT code for a urinalysis with microscopy is 81001. However, the question asks for the *most appropriate* code for the *patient’s condition* as documented, not the procedure performed. Therefore, the focus should be on the diagnosis. The documentation clearly points to cystitis. While other conditions might be considered in a differential diagnosis, the documented symptoms directly support cystitis. The absence of hematuria is also noted, which is crucial for selecting the correct subcategory within N30.0-. Therefore, N30.00 accurately reflects the documented condition. The explanation of why this is the correct choice involves understanding the ICD-10-CM coding conventions for genitourinary system diseases and the importance of precise documentation to select the most specific code. A thorough understanding of medical terminology, particularly terms related to the urinary system and inflammation, is essential. Furthermore, recognizing the distinction between a diagnosis code and a procedure code is fundamental for accurate medical billing and coding, a core competency at CMCB University. The correct approach involves analyzing the physician’s notes to identify the primary condition being treated and then locating the most specific ICD-10-CM code that reflects that condition, considering any qualifying details provided in the documentation.
-
Question 11 of 30
11. Question
A patient at CMCB University Medical Center presents with persistent epigastric pain and difficulty swallowing. Dr. Anya Sharma, a gastroenterologist, performs an esophagogastroduodenoscopy (EGD) to investigate these symptoms. The procedure involves the insertion of a flexible endoscope through the esophagus to visualize the upper gastrointestinal tract, including the stomach and duodenum. During the examination, Dr. Sharma identifies a small, irregular lesion in the gastric antrum and obtains a biopsy for further examination. Which CPT code accurately reflects the procedural service provided by Dr. Sharma, considering the diagnostic visualization and tissue sampling?
Correct
The scenario describes a patient presenting with symptoms indicative of a gastrointestinal issue. The physician performs a diagnostic procedure to visualize the upper digestive tract. The documentation notes the use of a flexible endoscope inserted through the esophagus to examine the stomach and duodenum. During the procedure, a biopsy of a suspicious lesion in the gastric antrum is taken for histopathological analysis. For coding purposes, the primary procedure is the esophagogastroduodenoscopy (EGD). The ICD-10-CM code for the diagnosis would depend on the findings of the biopsy and the physician’s final assessment. However, the question focuses on the procedural coding. The CPT code for an EGD with biopsy of the stomach is 43239. This code encompasses the visualization of the esophagus, stomach, and duodenum, and the procurement of tissue samples. The use of a flexible endoscope is inherent in this code. The location of the biopsy (gastric antrum) is a specific site within the stomach, and the code 43239 accurately reflects the service performed, including the biopsy. No additional modifiers are immediately apparent as necessary for this specific procedure without further context regarding payer or specific circumstances of the encounter. Therefore, the correct CPT code is 43239.
Incorrect
The scenario describes a patient presenting with symptoms indicative of a gastrointestinal issue. The physician performs a diagnostic procedure to visualize the upper digestive tract. The documentation notes the use of a flexible endoscope inserted through the esophagus to examine the stomach and duodenum. During the procedure, a biopsy of a suspicious lesion in the gastric antrum is taken for histopathological analysis. For coding purposes, the primary procedure is the esophagogastroduodenoscopy (EGD). The ICD-10-CM code for the diagnosis would depend on the findings of the biopsy and the physician’s final assessment. However, the question focuses on the procedural coding. The CPT code for an EGD with biopsy of the stomach is 43239. This code encompasses the visualization of the esophagus, stomach, and duodenum, and the procurement of tissue samples. The use of a flexible endoscope is inherent in this code. The location of the biopsy (gastric antrum) is a specific site within the stomach, and the code 43239 accurately reflects the service performed, including the biopsy. No additional modifiers are immediately apparent as necessary for this specific procedure without further context regarding payer or specific circumstances of the encounter. Therefore, the correct CPT code is 43239.
-
Question 12 of 30
12. Question
A patient is admitted to the hospital presenting with severe hematemesis and melena. The attending physician documents a history of epigastric pain and notes the presence of blood in the vomitus and dark, tarry stools. An esophagogastroduodenoscopy (EGD) is performed, revealing a bleeding duodenal ulcer. The ulcer is successfully treated with endoscopic cauterization. For CMCB University’s advanced coding program, which combination of ICD-10-CM and CPT codes most accurately reflects this patient’s encounter, considering the principal diagnosis and the therapeutic intervention performed?
Correct
The scenario describes a patient presenting with symptoms indicative of a gastrointestinal bleed. The physician documents “hematemesis” and “melena.” Hematemesis refers to vomiting blood, and melena refers to the passage of dark, tarry stools, which is a sign of gastrointestinal bleeding. The physician also orders an esophagogastroduodenoscopy (EGD) to visualize the upper gastrointestinal tract and identify the source of bleeding. During the EGD, a bleeding duodenal ulcer is identified and treated with cauterization. To accurately code this encounter for CMCB University’s curriculum, we need to consider the principal diagnosis and the procedure performed. The principal diagnosis is the condition established after study to be chiefly responsible for occasioning the admission of the patient to the hospital. In this case, the hematemesis and melena are symptoms of the underlying duodenal ulcer, which was identified as the cause of the bleeding. Therefore, the duodenal ulcer with hemorrhage is the principal diagnosis. The physician’s documentation supports the diagnosis of a duodenal ulcer with hemorrhage. The EGD procedure is a diagnostic and therapeutic intervention. The cauterization performed during the EGD is a crucial detail for coding. Considering ICD-10-CM coding, a duodenal ulcer with hemorrhage would be coded under category K26. The specific code for a duodenal ulcer with hemorrhage, unspecified, is K26.0. For CPT coding, the EGD procedure itself is coded. A diagnostic EGD with visualization of the esophagus, stomach, and duodenum is typically coded using 43235. However, since a bleeding duodenal ulcer was identified and treated with cauterization, a more specific code reflecting the therapeutic intervention is required. The CPT code for esophagogastroduodenoscopy, flexible, with endoscopic cauterization of bleeding lesion(s) in the esophagus, stomach, or duodenum is 43289. This code encompasses both the diagnostic visualization and the therapeutic cauterization. Therefore, the correct coding combination for this scenario, aligning with the principles taught at CMCB University, involves identifying the principal diagnosis of a bleeding duodenal ulcer and the appropriate CPT code for the therapeutic EGD. The explanation focuses on the diagnostic reasoning, the procedural intervention, and the application of both ICD-10-CM and CPT coding guidelines to arrive at the most accurate representation of the patient’s encounter. This demonstrates a nuanced understanding of how symptoms, diagnoses, and procedures are translated into billable codes, a core competency for CMCB University graduates.
Incorrect
The scenario describes a patient presenting with symptoms indicative of a gastrointestinal bleed. The physician documents “hematemesis” and “melena.” Hematemesis refers to vomiting blood, and melena refers to the passage of dark, tarry stools, which is a sign of gastrointestinal bleeding. The physician also orders an esophagogastroduodenoscopy (EGD) to visualize the upper gastrointestinal tract and identify the source of bleeding. During the EGD, a bleeding duodenal ulcer is identified and treated with cauterization. To accurately code this encounter for CMCB University’s curriculum, we need to consider the principal diagnosis and the procedure performed. The principal diagnosis is the condition established after study to be chiefly responsible for occasioning the admission of the patient to the hospital. In this case, the hematemesis and melena are symptoms of the underlying duodenal ulcer, which was identified as the cause of the bleeding. Therefore, the duodenal ulcer with hemorrhage is the principal diagnosis. The physician’s documentation supports the diagnosis of a duodenal ulcer with hemorrhage. The EGD procedure is a diagnostic and therapeutic intervention. The cauterization performed during the EGD is a crucial detail for coding. Considering ICD-10-CM coding, a duodenal ulcer with hemorrhage would be coded under category K26. The specific code for a duodenal ulcer with hemorrhage, unspecified, is K26.0. For CPT coding, the EGD procedure itself is coded. A diagnostic EGD with visualization of the esophagus, stomach, and duodenum is typically coded using 43235. However, since a bleeding duodenal ulcer was identified and treated with cauterization, a more specific code reflecting the therapeutic intervention is required. The CPT code for esophagogastroduodenoscopy, flexible, with endoscopic cauterization of bleeding lesion(s) in the esophagus, stomach, or duodenum is 43289. This code encompasses both the diagnostic visualization and the therapeutic cauterization. Therefore, the correct coding combination for this scenario, aligning with the principles taught at CMCB University, involves identifying the principal diagnosis of a bleeding duodenal ulcer and the appropriate CPT code for the therapeutic EGD. The explanation focuses on the diagnostic reasoning, the procedural intervention, and the application of both ICD-10-CM and CPT coding guidelines to arrive at the most accurate representation of the patient’s encounter. This demonstrates a nuanced understanding of how symptoms, diagnoses, and procedures are translated into billable codes, a core competency for CMCB University graduates.
-
Question 13 of 30
13. Question
A 68-year-old male, Mr. Aris Thorne, presents to the emergency department with sudden onset of severe substernal chest pain radiating to his left arm, accompanied by profuse sweating and shortness of breath. An electrocardiogram (ECG) reveals ST-segment elevation in leads II, III, and aVF. The physician’s assessment confirms an acute ST elevation myocardial infarction (STEMI) and notes a history of essential hypertension, well-controlled with medication. Considering the principles of ICD-10-CM coding for principal and secondary diagnoses as taught at CMCB University, what is the most appropriate coding sequence for Mr. Thorne’s encounter?
Correct
The scenario describes a patient presenting with symptoms indicative of an acute myocardial infarction (AMI). The physician’s documentation notes the presence of chest pain, diaphoresis, and ST-segment elevation on the electrocardiogram (ECG). The diagnosis of AMI is confirmed. For coding purposes, the primary diagnosis is AMI. The ICD-10-CM code for AMI is found by referencing the Alphabetic Index and then verifying in the Tabular List. Searching for “infarction, myocardial” leads to subterms like “acute” and “transmural” or “subendocardial.” Given the documentation of ST-segment elevation, which typically indicates a transmural infarction, the appropriate code category is I21. The specific code for ST elevation (STEMI) AMI is I21.3, ST elevation myocardial infarction. The documentation also mentions a history of hypertension, which is a co-existing condition. Hypertension is coded as I10, Essential (primary) hypertension. The guidelines for ICD-10-CM coding state that when a patient has hypertension and an AMI, and the hypertension is not stated as the cause of the AMI, the hypertension should be coded as a secondary diagnosis. Therefore, the correct coding sequence would be to list the AMI as the principal diagnosis followed by the hypertension. The question asks for the most appropriate coding sequence. The correct sequence prioritizes the acute condition requiring the most resources or the reason for the encounter.
Incorrect
The scenario describes a patient presenting with symptoms indicative of an acute myocardial infarction (AMI). The physician’s documentation notes the presence of chest pain, diaphoresis, and ST-segment elevation on the electrocardiogram (ECG). The diagnosis of AMI is confirmed. For coding purposes, the primary diagnosis is AMI. The ICD-10-CM code for AMI is found by referencing the Alphabetic Index and then verifying in the Tabular List. Searching for “infarction, myocardial” leads to subterms like “acute” and “transmural” or “subendocardial.” Given the documentation of ST-segment elevation, which typically indicates a transmural infarction, the appropriate code category is I21. The specific code for ST elevation (STEMI) AMI is I21.3, ST elevation myocardial infarction. The documentation also mentions a history of hypertension, which is a co-existing condition. Hypertension is coded as I10, Essential (primary) hypertension. The guidelines for ICD-10-CM coding state that when a patient has hypertension and an AMI, and the hypertension is not stated as the cause of the AMI, the hypertension should be coded as a secondary diagnosis. Therefore, the correct coding sequence would be to list the AMI as the principal diagnosis followed by the hypertension. The question asks for the most appropriate coding sequence. The correct sequence prioritizes the acute condition requiring the most resources or the reason for the encounter.
-
Question 14 of 30
14. Question
A patient presents to a primary care clinic at CMCB University Health System reporting burning during urination, a frequent urge to urinate, and a persistent feeling of needing to urinate. The physician’s notes indicate a urinalysis revealing the presence of leukocytes and nitrites. A pelvic examination was conducted to exclude other potential causes of the patient’s symptoms. Based on this documentation, which ICD-10-CM code best represents the patient’s primary diagnosis for billing and record-keeping purposes within the CMCB University Health System’s established coding guidelines?
Correct
The scenario describes a patient presenting with symptoms indicative of a urinary tract infection (UTI), specifically cystitis. The physician’s documentation notes dysuria, frequency, and urgency, along with a positive urinalysis for leukocytes and nitrites. The physician also performed a pelvic exam to rule out other causes of the symptoms. For coding purposes, the primary diagnosis is the UTI. ICD-10-CM code N39.0, “Urinary tract infection, site not specified,” is the most appropriate code for uncomplicated cystitis when the specific anatomical site within the urinary tract is not further detailed in the documentation. The pelvic exam, while a diagnostic procedure, does not alter the primary diagnosis code for the UTI itself. The documentation supports the presence of a UTI, and N39.0 accurately reflects this condition as the principal diagnosis. The other options are less specific or represent different conditions. For instance, R30.0 is for dysuria alone, which is a symptom, not the definitive diagnosis. N30.00 is for acute cystitis without hematuria, which is more specific but N39.0 is generally used for unspecified UTIs and is often the initial code assigned unless further specificity is clearly documented. N39.9 is for “Unspecified disorder of urinary system,” which is too broad. Therefore, N39.0 is the most accurate and encompassing code for the documented condition.
Incorrect
The scenario describes a patient presenting with symptoms indicative of a urinary tract infection (UTI), specifically cystitis. The physician’s documentation notes dysuria, frequency, and urgency, along with a positive urinalysis for leukocytes and nitrites. The physician also performed a pelvic exam to rule out other causes of the symptoms. For coding purposes, the primary diagnosis is the UTI. ICD-10-CM code N39.0, “Urinary tract infection, site not specified,” is the most appropriate code for uncomplicated cystitis when the specific anatomical site within the urinary tract is not further detailed in the documentation. The pelvic exam, while a diagnostic procedure, does not alter the primary diagnosis code for the UTI itself. The documentation supports the presence of a UTI, and N39.0 accurately reflects this condition as the principal diagnosis. The other options are less specific or represent different conditions. For instance, R30.0 is for dysuria alone, which is a symptom, not the definitive diagnosis. N30.00 is for acute cystitis without hematuria, which is more specific but N39.0 is generally used for unspecified UTIs and is often the initial code assigned unless further specificity is clearly documented. N39.9 is for “Unspecified disorder of urinary system,” which is too broad. Therefore, N39.0 is the most accurate and encompassing code for the documented condition.
-
Question 15 of 30
15. Question
A patient is admitted to the hospital with severe hematemesis and melena. The admitting physician’s assessment and plan document a diagnosis of “esophageal varices” as the cause of the bleeding. For a medical coder at CMCB University preparing the patient’s record for billing, which ICD-10-CM code best represents the principal diagnosis for this encounter, reflecting the underlying etiology of the patient’s symptoms?
Correct
The scenario describes a patient presenting with symptoms indicative of a gastrointestinal bleed. The physician documents “hematemesis” and “melena,” along with a diagnosis of “esophageal varices.” To accurately code this encounter for CMCB University’s curriculum, the coder must identify the principal diagnosis and any associated conditions that are relevant to the patient’s care and the services provided. Hematemesis refers to vomiting blood, and melena refers to black, tarry stools, both classic signs of upper gastrointestinal bleeding. Esophageal varices are swollen veins in the esophagus, often caused by liver disease, which can rupture and lead to severe bleeding. In ICD-10-CM, the coding guidelines emphasize sequencing the diagnosis that, after study, occasioned the admission to the hospital. In this case, the bleeding from the esophageal varices is the primary reason for the patient’s presentation and subsequent management. Therefore, the esophageal varices are the principal diagnosis. The symptoms of hematemesis and melena are manifestations of this underlying condition and are not coded separately when the underlying cause is known and documented. The ICD-10-CM code for esophageal varices, unspecified, is K65.8. However, the documentation specifies the cause of the bleeding is related to the varices. A more specific code is required. The guidelines for coding gastrointestinal hemorrhage state that if the site of the hemorrhage is known, the code for the hemorrhage should be sequenced first, followed by the code for the condition causing the hemorrhage. However, in this scenario, the physician has diagnosed the underlying cause (esophageal varices) and the symptoms are directly attributable to it. Therefore, the underlying condition should be sequenced first. Considering the specific etiology, esophageal varices are often linked to portal hypertension. The ICD-10-CM index would lead to codes for esophageal varices with or without bleeding, and further sub-classification based on the cause. For esophageal varices with bleeding, the appropriate code is K65.8, which represents “Esophageal varices.” While there are codes for gastrointestinal hemorrhage (K92.2), the guidelines suggest coding the underlying condition when it is the primary reason for admission and the cause of the symptoms. The physician’s diagnosis of esophageal varices as the cause of the bleeding necessitates coding the varices. Upon further review of ICD-10-CM guidelines and the index, esophageal varices are classified under diseases of the digestive system. The specific code for esophageal varices, unspecified, is K65.8. However, if the varices are specified as bleeding, the coding would reflect that. The symptoms of hematemesis and melena are integral to the diagnosis of bleeding esophageal varices and are not coded separately. The physician’s documentation clearly links the symptoms to the diagnosed condition. Therefore, the most accurate coding approach for CMCB University students to understand is to identify the principal diagnosis that explains the patient’s admission. In this case, the esophageal varices are the root cause of the bleeding symptoms. The correct ICD-10-CM code for esophageal varices, which are the underlying cause of the patient’s hematemesis and melena, is K65.8. This code accurately reflects the diagnosed condition that necessitated the patient’s encounter and treatment.
Incorrect
The scenario describes a patient presenting with symptoms indicative of a gastrointestinal bleed. The physician documents “hematemesis” and “melena,” along with a diagnosis of “esophageal varices.” To accurately code this encounter for CMCB University’s curriculum, the coder must identify the principal diagnosis and any associated conditions that are relevant to the patient’s care and the services provided. Hematemesis refers to vomiting blood, and melena refers to black, tarry stools, both classic signs of upper gastrointestinal bleeding. Esophageal varices are swollen veins in the esophagus, often caused by liver disease, which can rupture and lead to severe bleeding. In ICD-10-CM, the coding guidelines emphasize sequencing the diagnosis that, after study, occasioned the admission to the hospital. In this case, the bleeding from the esophageal varices is the primary reason for the patient’s presentation and subsequent management. Therefore, the esophageal varices are the principal diagnosis. The symptoms of hematemesis and melena are manifestations of this underlying condition and are not coded separately when the underlying cause is known and documented. The ICD-10-CM code for esophageal varices, unspecified, is K65.8. However, the documentation specifies the cause of the bleeding is related to the varices. A more specific code is required. The guidelines for coding gastrointestinal hemorrhage state that if the site of the hemorrhage is known, the code for the hemorrhage should be sequenced first, followed by the code for the condition causing the hemorrhage. However, in this scenario, the physician has diagnosed the underlying cause (esophageal varices) and the symptoms are directly attributable to it. Therefore, the underlying condition should be sequenced first. Considering the specific etiology, esophageal varices are often linked to portal hypertension. The ICD-10-CM index would lead to codes for esophageal varices with or without bleeding, and further sub-classification based on the cause. For esophageal varices with bleeding, the appropriate code is K65.8, which represents “Esophageal varices.” While there are codes for gastrointestinal hemorrhage (K92.2), the guidelines suggest coding the underlying condition when it is the primary reason for admission and the cause of the symptoms. The physician’s diagnosis of esophageal varices as the cause of the bleeding necessitates coding the varices. Upon further review of ICD-10-CM guidelines and the index, esophageal varices are classified under diseases of the digestive system. The specific code for esophageal varices, unspecified, is K65.8. However, if the varices are specified as bleeding, the coding would reflect that. The symptoms of hematemesis and melena are integral to the diagnosis of bleeding esophageal varices and are not coded separately. The physician’s documentation clearly links the symptoms to the diagnosed condition. Therefore, the most accurate coding approach for CMCB University students to understand is to identify the principal diagnosis that explains the patient’s admission. In this case, the esophageal varices are the root cause of the bleeding symptoms. The correct ICD-10-CM code for esophageal varices, which are the underlying cause of the patient’s hematemesis and melena, is K65.8. This code accurately reflects the diagnosed condition that necessitated the patient’s encounter and treatment.
-
Question 16 of 30
16. Question
A patient at CMCB University Medical Center presents with melena and hematochezia, accompanied by epigastric discomfort. The physician documents the encounter as “Patient presents with symptoms concerning for gastrointestinal bleeding. Rule out gastrointestinal bleed. Diagnostic colonoscopy performed.” Which ICD-10-CM code best represents the primary reason for this encounter and the subsequent diagnostic procedure, as per the physician’s documentation and the patient’s presenting signs?
Correct
The scenario describes a patient presenting with symptoms suggestive of a gastrointestinal bleed. The physician performs a diagnostic colonoscopy to identify the source of bleeding and potentially treat it. The colonoscopy itself is a procedure. The ICD-10-CM code for the *reason* for the encounter, which is the gastrointestinal bleeding, is R10.13, “Abdominal pain, epigastric.” However, the question asks for the *primary reason* for the colonoscopy as documented by the physician, which is the investigation of the bleeding. The physician’s documentation explicitly states “rule out gastrointestinal bleed.” Therefore, the most appropriate ICD-10-CM code to capture the *reason for the encounter* that necessitated the colonoscopy is K92.2, “Gastrointestinal hemorrhage, unspecified.” This code accurately reflects the physician’s diagnostic intent and the underlying condition being investigated. The colonoscopy procedure itself would be coded using CPT codes, but the question specifically asks for the ICD-10-CM diagnosis code that justifies the procedure. The other options represent conditions that are either unrelated to the presented symptoms or are less specific to the physician’s documented diagnostic objective. For instance, while abdominal pain might be a symptom, the primary driver for the colonoscopy is the suspected bleeding.
Incorrect
The scenario describes a patient presenting with symptoms suggestive of a gastrointestinal bleed. The physician performs a diagnostic colonoscopy to identify the source of bleeding and potentially treat it. The colonoscopy itself is a procedure. The ICD-10-CM code for the *reason* for the encounter, which is the gastrointestinal bleeding, is R10.13, “Abdominal pain, epigastric.” However, the question asks for the *primary reason* for the colonoscopy as documented by the physician, which is the investigation of the bleeding. The physician’s documentation explicitly states “rule out gastrointestinal bleed.” Therefore, the most appropriate ICD-10-CM code to capture the *reason for the encounter* that necessitated the colonoscopy is K92.2, “Gastrointestinal hemorrhage, unspecified.” This code accurately reflects the physician’s diagnostic intent and the underlying condition being investigated. The colonoscopy procedure itself would be coded using CPT codes, but the question specifically asks for the ICD-10-CM diagnosis code that justifies the procedure. The other options represent conditions that are either unrelated to the presented symptoms or are less specific to the physician’s documented diagnostic objective. For instance, while abdominal pain might be a symptom, the primary driver for the colonoscopy is the suspected bleeding.
-
Question 17 of 30
17. Question
A patient is admitted to CMCB University Medical Center with complaints of fatigue and dizziness. Upon examination, the physician notes the patient’s stools are dark and tarry, a condition documented as “melena.” The physician also observes bright red blood in the stool, recorded as “hematochezia.” To investigate the source of the bleeding, an esophagogastroduodenoscopy (EGD) is scheduled. Considering the clinical presentation and the planned diagnostic procedure, which ICD-10-CM code most accurately reflects the initial assessment and diagnostic focus for this patient at CMCB University?
Correct
The scenario describes a patient presenting with symptoms indicative of a gastrointestinal bleed. The physician documents “melena” and “hematochezia.” Melena refers to dark, tarry stools, typically caused by digested blood from an upper gastrointestinal source. Hematochezia refers to bright red blood in the stool, usually indicating bleeding from the lower gastrointestinal tract. The physician also orders an esophagogastroduodenoscopy (EGD), which is a procedure used to visualize the esophagus, stomach, and duodenum, commonly performed to diagnose upper gastrointestinal bleeding. Given the documentation of melena and the specific diagnostic procedure ordered (EGD), the primary focus of coding would be on identifying the underlying cause of the bleeding. The ICD-10-CM code for unspecified gastrointestinal hemorrhage, unspecified site, is K92.2. However, the physician’s documentation and the choice of EGD strongly suggest an upper GI bleed. Without further specific diagnostic findings from the EGD, the most appropriate ICD-10-CM code to capture the documented condition and the diagnostic approach, reflecting the potential for an upper GI bleed, is K29.00 (Gastritis, unspecified, with bleeding, without obstruction). This code accurately reflects the presence of bleeding associated with a gastric condition, which is a common source for melena and is directly investigated by an EGD. While other codes might represent bleeding, K29.00 best aligns with the clinical picture presented and the diagnostic pathway chosen by the physician at CMCB University’s affiliated teaching hospital, emphasizing the importance of linking documentation to the most specific and relevant diagnostic code.
Incorrect
The scenario describes a patient presenting with symptoms indicative of a gastrointestinal bleed. The physician documents “melena” and “hematochezia.” Melena refers to dark, tarry stools, typically caused by digested blood from an upper gastrointestinal source. Hematochezia refers to bright red blood in the stool, usually indicating bleeding from the lower gastrointestinal tract. The physician also orders an esophagogastroduodenoscopy (EGD), which is a procedure used to visualize the esophagus, stomach, and duodenum, commonly performed to diagnose upper gastrointestinal bleeding. Given the documentation of melena and the specific diagnostic procedure ordered (EGD), the primary focus of coding would be on identifying the underlying cause of the bleeding. The ICD-10-CM code for unspecified gastrointestinal hemorrhage, unspecified site, is K92.2. However, the physician’s documentation and the choice of EGD strongly suggest an upper GI bleed. Without further specific diagnostic findings from the EGD, the most appropriate ICD-10-CM code to capture the documented condition and the diagnostic approach, reflecting the potential for an upper GI bleed, is K29.00 (Gastritis, unspecified, with bleeding, without obstruction). This code accurately reflects the presence of bleeding associated with a gastric condition, which is a common source for melena and is directly investigated by an EGD. While other codes might represent bleeding, K29.00 best aligns with the clinical picture presented and the diagnostic pathway chosen by the physician at CMCB University’s affiliated teaching hospital, emphasizing the importance of linking documentation to the most specific and relevant diagnostic code.
-
Question 18 of 30
18. Question
A patient presents to their physician at CMCB University Medical Center with complaints of dark, tarry stools and generalized weakness, suggestive of a gastrointestinal bleed. A diagnostic colonoscopy is performed to identify the source of the bleeding. During the procedure, a 1.5 cm sessile polyp is visualized in the sigmoid colon and is successfully removed using snare cautery. The physician also documents a separate, brief encounter for the management of the patient’s essential hypertension. Which of the following coding and diagnostic combinations most accurately reflects the services rendered during the colonoscopy portion of the patient’s visit, adhering to CMCB University’s rigorous academic standards for documentation and coding accuracy?
Correct
The scenario describes a patient presenting with symptoms indicative of a gastrointestinal bleed. The physician performs a diagnostic colonoscopy to identify the source of bleeding. During the procedure, a polyp is identified and removed via snare cautery. The physician also documents a separate, unrelated encounter for a routine follow-up of hypertension. For the colonoscopy with polyp removal, the primary procedure is the colonoscopy itself, which is coded using CPT. The diagnostic colonoscopy to visualize the colon is coded as 45378 (Colonoscopy, flexible, sigmoidoscopy and/or proctosigmoidoscopy; with colonoscopy and endoscopic이션 of the colon). The polyp removal via snare cautery is an additional service performed during the colonoscopy. According to CPT guidelines, when a diagnostic colonoscopy is performed and a therapeutic intervention like polyp removal is done, the diagnostic code is reported along with the appropriate add-on code for the therapeutic service. In this case, the polyp removal using snare cautery is coded as 45385 (Colonoscopy, flexible, sigmoidoscopy and/or proctosigmoidoscopy; with endoscopic이션 of the colon; with snare- διάμετρος cautery). The physician also documented a separate encounter for hypertension follow-up. This is a distinct service and should be coded separately. Evaluation and Management (E/M) codes are used for these encounters. Given the physician is managing a chronic condition (hypertension) and performing a follow-up visit, an appropriate E/M code would be selected based on the level of medical decision making or time spent. For the purpose of this question, we will assume a moderate level of medical decision making, which could correspond to a code like 99214 (Office or other outpatient visit for the evaluation and management of an established patient, which requires at least 2 of 3 key components: a detailed history; a detailed examination; medical decision making of moderate complexity). The ICD-10-CM codes would reflect the diagnoses. The gastrointestinal bleed would be coded as K92.2 (Gastrointestinal hemorrhage, unspecified). The polyp found during the colonoscopy would be coded as K63.5 (Polyp of colon). The hypertension would be coded as I10 (Essential (primary) hypertension). When reporting these services, the primary procedure (colonoscopy with polyp removal) is listed first, followed by the E/M service for hypertension management. The ICD-10-CM codes are linked to the respective CPT codes. The colonoscopy and polyp removal (45378 and 45385) would be linked to K92.2 and K63.5. The hypertension follow-up (99214) would be linked to I10. The question asks for the most appropriate coding sequence and diagnosis linkage for the colonoscopy procedure and polyp removal, considering the documentation. The colonoscopy is diagnostic, and a polyp is found and removed. The appropriate CPT codes are 45378 for the diagnostic colonoscopy and 45385 for the snare cautery removal of the polyp. These procedures are performed to investigate the gastrointestinal bleed (K92.2) and to treat the identified polyp (K63.5). Therefore, the correct coding sequence and linkage involves reporting both CPT codes for the colonoscopy and polyp removal, with the primary diagnosis being the reason for the colonoscopy (GI bleed) and the secondary diagnosis being the finding during the procedure (polyp). The correct answer reflects the primary procedure (colonoscopy), the therapeutic intervention (polyp removal), and the associated diagnoses that justify both the diagnostic exploration and the therapeutic action. The inclusion of the hypertension management is a separate encounter and not directly related to the colonoscopy coding itself, though it would be billed as a distinct service. The question specifically focuses on the colonoscopy scenario.
Incorrect
The scenario describes a patient presenting with symptoms indicative of a gastrointestinal bleed. The physician performs a diagnostic colonoscopy to identify the source of bleeding. During the procedure, a polyp is identified and removed via snare cautery. The physician also documents a separate, unrelated encounter for a routine follow-up of hypertension. For the colonoscopy with polyp removal, the primary procedure is the colonoscopy itself, which is coded using CPT. The diagnostic colonoscopy to visualize the colon is coded as 45378 (Colonoscopy, flexible, sigmoidoscopy and/or proctosigmoidoscopy; with colonoscopy and endoscopic이션 of the colon). The polyp removal via snare cautery is an additional service performed during the colonoscopy. According to CPT guidelines, when a diagnostic colonoscopy is performed and a therapeutic intervention like polyp removal is done, the diagnostic code is reported along with the appropriate add-on code for the therapeutic service. In this case, the polyp removal using snare cautery is coded as 45385 (Colonoscopy, flexible, sigmoidoscopy and/or proctosigmoidoscopy; with endoscopic이션 of the colon; with snare- διάμετρος cautery). The physician also documented a separate encounter for hypertension follow-up. This is a distinct service and should be coded separately. Evaluation and Management (E/M) codes are used for these encounters. Given the physician is managing a chronic condition (hypertension) and performing a follow-up visit, an appropriate E/M code would be selected based on the level of medical decision making or time spent. For the purpose of this question, we will assume a moderate level of medical decision making, which could correspond to a code like 99214 (Office or other outpatient visit for the evaluation and management of an established patient, which requires at least 2 of 3 key components: a detailed history; a detailed examination; medical decision making of moderate complexity). The ICD-10-CM codes would reflect the diagnoses. The gastrointestinal bleed would be coded as K92.2 (Gastrointestinal hemorrhage, unspecified). The polyp found during the colonoscopy would be coded as K63.5 (Polyp of colon). The hypertension would be coded as I10 (Essential (primary) hypertension). When reporting these services, the primary procedure (colonoscopy with polyp removal) is listed first, followed by the E/M service for hypertension management. The ICD-10-CM codes are linked to the respective CPT codes. The colonoscopy and polyp removal (45378 and 45385) would be linked to K92.2 and K63.5. The hypertension follow-up (99214) would be linked to I10. The question asks for the most appropriate coding sequence and diagnosis linkage for the colonoscopy procedure and polyp removal, considering the documentation. The colonoscopy is diagnostic, and a polyp is found and removed. The appropriate CPT codes are 45378 for the diagnostic colonoscopy and 45385 for the snare cautery removal of the polyp. These procedures are performed to investigate the gastrointestinal bleed (K92.2) and to treat the identified polyp (K63.5). Therefore, the correct coding sequence and linkage involves reporting both CPT codes for the colonoscopy and polyp removal, with the primary diagnosis being the reason for the colonoscopy (GI bleed) and the secondary diagnosis being the finding during the procedure (polyp). The correct answer reflects the primary procedure (colonoscopy), the therapeutic intervention (polyp removal), and the associated diagnoses that justify both the diagnostic exploration and the therapeutic action. The inclusion of the hypertension management is a separate encounter and not directly related to the colonoscopy coding itself, though it would be billed as a distinct service. The question specifically focuses on the colonoscopy scenario.
-
Question 19 of 30
19. Question
A patient is admitted to the hospital with severe abdominal pain and signs of significant blood loss. The physician’s progress note details the following: “Patient reports vomiting bright red blood (hematemesis) and has passed dark, tarry stools (melena) for the past 24 hours, consistent with a significant upper gastrointestinal hemorrhage.” Considering the documentation provided and the principles of ICD-10-CM coding as taught at CMCB University, what is the most appropriate coding approach for this clinical presentation?
Correct
The scenario describes a patient presenting with symptoms indicative of a gastrointestinal bleed. The physician documents “hematemesis” and “melena.” Hematemesis refers to the vomiting of blood, and melena refers to the passage of dark, tarry stools, which is typically caused by digested blood. In ICD-10-CM, the primary diagnosis for hematemesis is K92.0. The documentation also specifies “melena,” which is coded as K92.1. When multiple conditions are documented that are related to the same physiological process or anatomical site, and one is more specific or represents a consequence of the other, the coder must determine the principal diagnosis. However, in this case, both hematemesis and melena are distinct manifestations of a gastrointestinal bleed, and ICD-10-CM guidelines direct coders to assign separate codes for each if they are documented. Therefore, the most accurate coding would include both K92.0 for hematemesis and K92.1 for melena. The question asks for the most appropriate coding approach given the documentation. The correct approach involves identifying and assigning the ICD-10-CM codes that precisely reflect the documented clinical conditions. This demonstrates an understanding of how to translate clinical terminology into standardized codes, a fundamental skill for a medical coder at CMCB University. Accurate coding ensures proper reimbursement and contributes to the integrity of patient health records, aligning with the university’s emphasis on scholarly principles and ethical requirements.
Incorrect
The scenario describes a patient presenting with symptoms indicative of a gastrointestinal bleed. The physician documents “hematemesis” and “melena.” Hematemesis refers to the vomiting of blood, and melena refers to the passage of dark, tarry stools, which is typically caused by digested blood. In ICD-10-CM, the primary diagnosis for hematemesis is K92.0. The documentation also specifies “melena,” which is coded as K92.1. When multiple conditions are documented that are related to the same physiological process or anatomical site, and one is more specific or represents a consequence of the other, the coder must determine the principal diagnosis. However, in this case, both hematemesis and melena are distinct manifestations of a gastrointestinal bleed, and ICD-10-CM guidelines direct coders to assign separate codes for each if they are documented. Therefore, the most accurate coding would include both K92.0 for hematemesis and K92.1 for melena. The question asks for the most appropriate coding approach given the documentation. The correct approach involves identifying and assigning the ICD-10-CM codes that precisely reflect the documented clinical conditions. This demonstrates an understanding of how to translate clinical terminology into standardized codes, a fundamental skill for a medical coder at CMCB University. Accurate coding ensures proper reimbursement and contributes to the integrity of patient health records, aligning with the university’s emphasis on scholarly principles and ethical requirements.
-
Question 20 of 30
20. Question
A patient presents to CMCB University Hospital for a scheduled laparoscopic cholecystectomy due to recurrent biliary colic. During the procedure, an intraoperative cholangiogram is performed to assess the common bile duct for potential stones. The surgeon’s operative report notes the successful removal of the gallbladder and the absence of common bile duct stones identified during the cholangiogram. Based on the provided information and adhering to the principles of accurate ICD-10-CM coding as taught at CMCB University, which of the following codes best represents the primary diagnosis for this encounter?
Correct
The scenario describes a patient undergoing a complex surgical procedure, a laparoscopic cholecystectomy, with an intraoperative cholangiogram. The key to accurate ICD-10-CM coding lies in identifying the primary reason for the encounter and any significant co-existing conditions or complications that affect patient care. The patient’s history of symptomatic cholelithiasis (gallstones) is the primary diagnosis driving the surgical intervention. Therefore, K80.20, Calculus of gallbladder without cholecystitis, unspecified, is the appropriate code for the underlying condition. The operative report details the laparoscopic cholecystectomy, which is a procedure to remove the gallbladder. The intraoperative cholangiogram, performed to visualize the bile ducts, is an integral part of the surgical management of cholelithiasis when complications like common bile duct stones are suspected or confirmed. While the cholangiogram itself is a procedure, the ICD-10-CM code reflects the *condition* being treated. The documentation does not explicitly state a complication like cholangitis or pancreatitis, which would necessitate additional codes. The focus remains on the symptomatic gallstones leading to the gallbladder removal. Therefore, K80.20 accurately captures the primary medical necessity for the encounter as presented in the case for CMCB University’s advanced coding curriculum, emphasizing the principle of coding the condition that occasioned the admission or encounter.
Incorrect
The scenario describes a patient undergoing a complex surgical procedure, a laparoscopic cholecystectomy, with an intraoperative cholangiogram. The key to accurate ICD-10-CM coding lies in identifying the primary reason for the encounter and any significant co-existing conditions or complications that affect patient care. The patient’s history of symptomatic cholelithiasis (gallstones) is the primary diagnosis driving the surgical intervention. Therefore, K80.20, Calculus of gallbladder without cholecystitis, unspecified, is the appropriate code for the underlying condition. The operative report details the laparoscopic cholecystectomy, which is a procedure to remove the gallbladder. The intraoperative cholangiogram, performed to visualize the bile ducts, is an integral part of the surgical management of cholelithiasis when complications like common bile duct stones are suspected or confirmed. While the cholangiogram itself is a procedure, the ICD-10-CM code reflects the *condition* being treated. The documentation does not explicitly state a complication like cholangitis or pancreatitis, which would necessitate additional codes. The focus remains on the symptomatic gallstones leading to the gallbladder removal. Therefore, K80.20 accurately captures the primary medical necessity for the encounter as presented in the case for CMCB University’s advanced coding curriculum, emphasizing the principle of coding the condition that occasioned the admission or encounter.
-
Question 21 of 30
21. Question
A patient presents to the clinic at CMCB University with symptoms of dysuria and increased urinary frequency. The physician orders a urinalysis, which reveals the presence of leukocytes and nitrites. A subsequent urine culture and sensitivity test identifies *Escherichia coli* as the causative organism, demonstrating susceptibility to nitrofurantoin but resistance to ampicillin. Which combination of ICD-10-CM and CPT codes accurately reflects the physician’s diagnosis and the laboratory procedures performed?
Correct
The scenario describes a patient presenting with symptoms indicative of a urinary tract infection (UTI). The physician performs a urinalysis and a urine culture and sensitivity (C&S) test. The urinalysis reveals leukocytes and nitrites, which are common indicators of a UTI. The urine culture identifies *Escherichia coli* (E. coli) as the causative agent, and the sensitivity testing shows that this strain of E. coli is susceptible to nitrofurantoin but resistant to ampicillin. Based on this clinical information, the medical coder must select the most accurate ICD-10-CM diagnosis code and the appropriate CPT code for the procedures performed. For the diagnosis, the presence of leukocytes and nitrites in the urinalysis, along with the positive culture identifying E. coli, strongly supports a diagnosis of a bacterial UTI. ICD-10-CM code N39.0, “Urinary tract infection, site not specified,” is the most appropriate code as the specific location of the infection within the urinary tract (e.g., bladder, kidney) is not definitively stated in the provided information, and the focus is on the general presence of infection. While other codes might exist for specific sites, N39.0 encompasses the overall condition described. For the procedures, a urinalysis is typically reported with CPT code 81001, “Urinalysis, automated, with microscopy.” However, the description mentions “leukocytes and nitrites,” which are often part of a dipstick or automated chemical analysis. CPT code 81000, “Urinalysis, with microscopy, manual,” or 81002, “Urinalysis, without microscopy, automated,” could be considered depending on the exact methodology. Given the mention of specific findings (leukocytes, nitrites) often derived from automated dipstick analysis, and the absence of explicit mention of manual microscopy, 81002 is a strong candidate. The urine culture and sensitivity test is reported with CPT code 87086, “Culture and sensitivity, bacterial; urine.” This code accurately reflects the laboratory procedure of identifying bacteria in a urine sample and determining its susceptibility to various antibiotics. Therefore, the correct combination of codes for this scenario, reflecting the diagnosis and the procedures performed, is N39.0 for the UTI and 87086 for the urine culture and sensitivity. The urinalysis code is not explicitly requested in the options, but the core diagnostic and procedural elements are captured. The question focuses on the most critical coding elements for reimbursement and medical record accuracy at CMCB University’s rigorous standards.
Incorrect
The scenario describes a patient presenting with symptoms indicative of a urinary tract infection (UTI). The physician performs a urinalysis and a urine culture and sensitivity (C&S) test. The urinalysis reveals leukocytes and nitrites, which are common indicators of a UTI. The urine culture identifies *Escherichia coli* (E. coli) as the causative agent, and the sensitivity testing shows that this strain of E. coli is susceptible to nitrofurantoin but resistant to ampicillin. Based on this clinical information, the medical coder must select the most accurate ICD-10-CM diagnosis code and the appropriate CPT code for the procedures performed. For the diagnosis, the presence of leukocytes and nitrites in the urinalysis, along with the positive culture identifying E. coli, strongly supports a diagnosis of a bacterial UTI. ICD-10-CM code N39.0, “Urinary tract infection, site not specified,” is the most appropriate code as the specific location of the infection within the urinary tract (e.g., bladder, kidney) is not definitively stated in the provided information, and the focus is on the general presence of infection. While other codes might exist for specific sites, N39.0 encompasses the overall condition described. For the procedures, a urinalysis is typically reported with CPT code 81001, “Urinalysis, automated, with microscopy.” However, the description mentions “leukocytes and nitrites,” which are often part of a dipstick or automated chemical analysis. CPT code 81000, “Urinalysis, with microscopy, manual,” or 81002, “Urinalysis, without microscopy, automated,” could be considered depending on the exact methodology. Given the mention of specific findings (leukocytes, nitrites) often derived from automated dipstick analysis, and the absence of explicit mention of manual microscopy, 81002 is a strong candidate. The urine culture and sensitivity test is reported with CPT code 87086, “Culture and sensitivity, bacterial; urine.” This code accurately reflects the laboratory procedure of identifying bacteria in a urine sample and determining its susceptibility to various antibiotics. Therefore, the correct combination of codes for this scenario, reflecting the diagnosis and the procedures performed, is N39.0 for the UTI and 87086 for the urine culture and sensitivity. The urinalysis code is not explicitly requested in the options, but the core diagnostic and procedural elements are captured. The question focuses on the most critical coding elements for reimbursement and medical record accuracy at CMCB University’s rigorous standards.
-
Question 22 of 30
22. Question
A patient is admitted to the hospital reporting significant hematemesis and melena. An esophagogastroduodenoscopy (EGD) is performed, revealing an acute gastric ulcer with active bleeding. During the procedure, the physician obtains a biopsy of the ulcer and successfully cauterizes the bleeding site. Considering the documentation and the procedures performed, what is the most accurate and comprehensive coding combination for this encounter, as would be assessed in a clinical scenario at CMCB University?
Correct
The scenario describes a patient presenting with symptoms of a gastrointestinal bleed. The physician performs an esophagogastroduodenoscopy (EGD) with biopsy and cauterization of a bleeding ulcer. To accurately code this encounter for CMCB University’s curriculum, we must identify the appropriate ICD-10-CM diagnosis code and CPT procedure codes. For the diagnosis, the patient has a bleeding ulcer of the stomach. The ICD-10-CM code K25.0, Gastric ulcer, acute, with hemorrhage, accurately reflects this condition. This code specifically addresses an acute gastric ulcer that is actively bleeding, which aligns with the documentation of “hematemesis” and “melena.” For the procedures, the physician performed an EGD, a biopsy during the EGD, and cauterization of the bleeding site. The CPT code for an EGD with a diagnostic biopsy is 43239 (Esophagogastroduodenoscopy, flexible, with biopsy, single or multiple). The CPT code for cauterization of a bleeding ulcer during an EGD is 43289 (Esophagogastroduodenoscopy, flexible, with control of intra-abdominal hemorrhage, via natural or artificial opening, endoscopic). When multiple procedures are performed during the same operative session, modifiers are often appended to indicate this. However, for these specific codes, the National Correct Coding Initiative (NCCI) edits do not bundle them, and both are typically reported. The question asks for the most appropriate *combination* of codes that represents the entire clinical encounter. Therefore, the correct coding would involve the diagnosis code K25.0 and the CPT codes 43239 and 43289. The explanation focuses on the precise diagnostic code for a bleeding gastric ulcer and the specific CPT codes for the endoscopic procedures performed, including biopsy and hemorrhage control. It highlights the importance of selecting codes that capture the acuity of the condition and the interventions undertaken, aligning with the rigorous standards of medical coding and billing taught at CMCB University. Understanding the nuances of ICD-10-CM and CPT coding, including the relationship between diagnoses and procedures, is fundamental to accurate reimbursement and patient record integrity, core competencies for graduates of CMCB University.
Incorrect
The scenario describes a patient presenting with symptoms of a gastrointestinal bleed. The physician performs an esophagogastroduodenoscopy (EGD) with biopsy and cauterization of a bleeding ulcer. To accurately code this encounter for CMCB University’s curriculum, we must identify the appropriate ICD-10-CM diagnosis code and CPT procedure codes. For the diagnosis, the patient has a bleeding ulcer of the stomach. The ICD-10-CM code K25.0, Gastric ulcer, acute, with hemorrhage, accurately reflects this condition. This code specifically addresses an acute gastric ulcer that is actively bleeding, which aligns with the documentation of “hematemesis” and “melena.” For the procedures, the physician performed an EGD, a biopsy during the EGD, and cauterization of the bleeding site. The CPT code for an EGD with a diagnostic biopsy is 43239 (Esophagogastroduodenoscopy, flexible, with biopsy, single or multiple). The CPT code for cauterization of a bleeding ulcer during an EGD is 43289 (Esophagogastroduodenoscopy, flexible, with control of intra-abdominal hemorrhage, via natural or artificial opening, endoscopic). When multiple procedures are performed during the same operative session, modifiers are often appended to indicate this. However, for these specific codes, the National Correct Coding Initiative (NCCI) edits do not bundle them, and both are typically reported. The question asks for the most appropriate *combination* of codes that represents the entire clinical encounter. Therefore, the correct coding would involve the diagnosis code K25.0 and the CPT codes 43239 and 43289. The explanation focuses on the precise diagnostic code for a bleeding gastric ulcer and the specific CPT codes for the endoscopic procedures performed, including biopsy and hemorrhage control. It highlights the importance of selecting codes that capture the acuity of the condition and the interventions undertaken, aligning with the rigorous standards of medical coding and billing taught at CMCB University. Understanding the nuances of ICD-10-CM and CPT coding, including the relationship between diagnoses and procedures, is fundamental to accurate reimbursement and patient record integrity, core competencies for graduates of CMCB University.
-
Question 23 of 30
23. Question
A patient is admitted to the hospital with complaints of vomiting blood and passing dark, tarry stools. The physician’s initial assessment notes “hematemesis” and “melena” in the progress notes. Considering the principles of accurate ICD-10-CM coding as emphasized in the Medical Coder and Biller program at CMCB University, which diagnostic code best represents the documented clinical findings for this patient’s presentation, assuming no further specific etiology for the bleeding is identified in the initial documentation?
Correct
The scenario describes a patient presenting with symptoms suggestive of a gastrointestinal bleed. The physician documents “hematemesis” and “melena.” Hematemesis refers to the vomiting of blood, and melena refers to the passage of dark, tarry stools, which indicates digested blood. In ICD-10-CM, gastrointestinal hemorrhages are categorized based on their location. For upper gastrointestinal bleeding, which includes conditions like esophageal varices, peptic ulcers, and gastritis, the relevant codes are found in the K20-K31 block. Specifically, K92.2 is the code for “Gastrointestinal hemorrhage, unspecified.” However, the documentation provides more specific clinical information that allows for a more precise coding. “Hematemesis” points to an upper GI source. While melena also indicates a GI bleed, the combination of hematemesis and melena strongly suggests an upper GI origin. The ICD-10-CM Official Guidelines for Coding and Reporting state that when a definitive diagnosis is documented, it should be used instead of symptoms. Given the symptoms, a more specific code for upper gastrointestinal hemorrhage would be appropriate if the documentation supported it. However, without a definitive diagnosis of the *cause* of the bleed (e.g., peptic ulcer, gastritis), the most accurate code reflecting the documented *condition* of gastrointestinal hemorrhage, considering the symptoms presented, is K92.2. This code is used when the site or cause of the gastrointestinal hemorrhage is not specified, or when both upper and lower GI bleeding are present without further specification. The explanation emphasizes the importance of using the most specific code available based on the physician’s documentation, aligning with the principles of accurate medical coding taught at CMCB University, which stresses the need to translate clinical documentation into precise diagnostic codes for proper reimbursement and data analysis. Understanding the nuances of ICD-10-CM coding for various body systems, like the digestive system in this case, is a cornerstone of the curriculum.
Incorrect
The scenario describes a patient presenting with symptoms suggestive of a gastrointestinal bleed. The physician documents “hematemesis” and “melena.” Hematemesis refers to the vomiting of blood, and melena refers to the passage of dark, tarry stools, which indicates digested blood. In ICD-10-CM, gastrointestinal hemorrhages are categorized based on their location. For upper gastrointestinal bleeding, which includes conditions like esophageal varices, peptic ulcers, and gastritis, the relevant codes are found in the K20-K31 block. Specifically, K92.2 is the code for “Gastrointestinal hemorrhage, unspecified.” However, the documentation provides more specific clinical information that allows for a more precise coding. “Hematemesis” points to an upper GI source. While melena also indicates a GI bleed, the combination of hematemesis and melena strongly suggests an upper GI origin. The ICD-10-CM Official Guidelines for Coding and Reporting state that when a definitive diagnosis is documented, it should be used instead of symptoms. Given the symptoms, a more specific code for upper gastrointestinal hemorrhage would be appropriate if the documentation supported it. However, without a definitive diagnosis of the *cause* of the bleed (e.g., peptic ulcer, gastritis), the most accurate code reflecting the documented *condition* of gastrointestinal hemorrhage, considering the symptoms presented, is K92.2. This code is used when the site or cause of the gastrointestinal hemorrhage is not specified, or when both upper and lower GI bleeding are present without further specification. The explanation emphasizes the importance of using the most specific code available based on the physician’s documentation, aligning with the principles of accurate medical coding taught at CMCB University, which stresses the need to translate clinical documentation into precise diagnostic codes for proper reimbursement and data analysis. Understanding the nuances of ICD-10-CM coding for various body systems, like the digestive system in this case, is a cornerstone of the curriculum.
-
Question 24 of 30
24. Question
During a patient encounter at CMCB University Medical Center, Dr. Anya Sharma documents “hematemesis” and “melena” for a patient experiencing upper gastrointestinal distress. Following a diagnostic esophagogastroduodenoscopy (EGD) with biopsy, Dr. Sharma identifies a bleeding duodenal ulcer. Which combination of CPT and ICD-10-CM codes most accurately reflects this clinical scenario for billing purposes at CMCB University Medical Center?
Correct
The scenario describes a patient presenting with symptoms indicative of a gastrointestinal bleed. The physician documents “hematemesis” and “melena.” Hematemesis refers to vomiting blood, and melena refers to the passage of dark, tarry stools, which is indicative of digested blood. The physician also performs a diagnostic esophagogastroduodenoscopy (EGD) to visualize the upper gastrointestinal tract and identifies a bleeding duodenal ulcer. The EGD procedure is coded using CPT. The diagnostic EGD, which includes visualization and biopsy, falls under the CPT code range for upper gastrointestinal endoscopy. Specifically, a diagnostic EGD with biopsy is typically represented by a code from the 43235-43259 range. Given the identification of a bleeding ulcer, a modifier may be appended to indicate the complexity or specific findings, but the core procedure is the EGD. The diagnosis of a bleeding duodenal ulcer is coded using ICD-10-CM. Duodenal ulcers are classified under K26. A bleeding duodenal ulcer is specified by K26.4. Therefore, the correct coding sequence involves identifying the appropriate CPT code for the diagnostic EGD with biopsy and the ICD-10-CM code for the bleeding duodenal ulcer. The explanation focuses on the accurate application of both coding systems to reflect the physician’s documentation and the patient’s condition, a core competency for graduates of CMCB University’s Medical Coder and Biller program. Understanding the relationship between clinical documentation, procedural coding, and diagnostic coding is paramount for accurate reimbursement and compliance, aligning with the program’s emphasis on scholarly principles and ethical requirements.
Incorrect
The scenario describes a patient presenting with symptoms indicative of a gastrointestinal bleed. The physician documents “hematemesis” and “melena.” Hematemesis refers to vomiting blood, and melena refers to the passage of dark, tarry stools, which is indicative of digested blood. The physician also performs a diagnostic esophagogastroduodenoscopy (EGD) to visualize the upper gastrointestinal tract and identifies a bleeding duodenal ulcer. The EGD procedure is coded using CPT. The diagnostic EGD, which includes visualization and biopsy, falls under the CPT code range for upper gastrointestinal endoscopy. Specifically, a diagnostic EGD with biopsy is typically represented by a code from the 43235-43259 range. Given the identification of a bleeding ulcer, a modifier may be appended to indicate the complexity or specific findings, but the core procedure is the EGD. The diagnosis of a bleeding duodenal ulcer is coded using ICD-10-CM. Duodenal ulcers are classified under K26. A bleeding duodenal ulcer is specified by K26.4. Therefore, the correct coding sequence involves identifying the appropriate CPT code for the diagnostic EGD with biopsy and the ICD-10-CM code for the bleeding duodenal ulcer. The explanation focuses on the accurate application of both coding systems to reflect the physician’s documentation and the patient’s condition, a core competency for graduates of CMCB University’s Medical Coder and Biller program. Understanding the relationship between clinical documentation, procedural coding, and diagnostic coding is paramount for accurate reimbursement and compliance, aligning with the program’s emphasis on scholarly principles and ethical requirements.
-
Question 25 of 30
25. Question
During a patient encounter at CMCB University’s affiliated teaching hospital, a physician documents the following: “Patient presented with severe hematemesis and melena. Physical examination revealed signs of portal hypertension. Imaging confirmed the presence of esophageal varices. Treatment focused on controlling the active hemorrhage. Patient’s history includes chronic alcoholic cirrhosis.” Which ICD-10-CM code accurately represents the principal diagnosis for this encounter, reflecting the primary reason for the patient’s admission and management?
Correct
The scenario describes a patient presenting with symptoms indicative of a gastrointestinal bleed, specifically hematemesis and melena. The physician documents a diagnosis of “Esophageal varices with hemorrhage.” To accurately code this encounter for CMCB University’s curriculum, we must identify the primary diagnosis and any associated conditions that require coding. The ICD-10-CM coding guidelines emphasize coding the condition chiefly responsible for the encounter. In this case, the hemorrhage from esophageal varices is the primary reason for the patient’s admission and treatment. Esophageal varices are abnormal, enlarged veins in the esophagus, often caused by liver disease. Hemorrhage from these varices is a serious complication. According to ICD-10-CM, esophageal varices are classified under K76.6 (Esophageal varices). However, the guidelines for coding hemorrhage state that if the hemorrhage is the principal diagnosis, it should be coded first. The specific code for esophageal varices with hemorrhage is I85.01 (Esophageal varices with hemorrhage). This code directly captures both the underlying condition (esophageal varices) and the acute event (hemorrhage). The physician’s documentation clearly links the hemorrhage to the esophageal varices. Therefore, I85.01 is the most appropriate principal diagnosis code. The documentation also mentions that the patient has a history of alcoholic cirrhosis, which is a common etiology for esophageal varices. While cirrhosis is a significant underlying condition, the immediate reason for the encounter is the bleeding. ICD-10-CM guidelines generally direct coders to code the condition that occasions the admission. In this instance, the hemorrhage is the acute event driving the medical necessity for the encounter. If the cirrhosis were the primary focus of treatment during this specific encounter, or if it were a more severe manifestation requiring specific management beyond the bleeding itself, it might be coded differently or as a secondary diagnosis. However, based on the provided documentation, the hemorrhage from the varices is the principal concern. Therefore, the correct coding sequence begins with I85.01.
Incorrect
The scenario describes a patient presenting with symptoms indicative of a gastrointestinal bleed, specifically hematemesis and melena. The physician documents a diagnosis of “Esophageal varices with hemorrhage.” To accurately code this encounter for CMCB University’s curriculum, we must identify the primary diagnosis and any associated conditions that require coding. The ICD-10-CM coding guidelines emphasize coding the condition chiefly responsible for the encounter. In this case, the hemorrhage from esophageal varices is the primary reason for the patient’s admission and treatment. Esophageal varices are abnormal, enlarged veins in the esophagus, often caused by liver disease. Hemorrhage from these varices is a serious complication. According to ICD-10-CM, esophageal varices are classified under K76.6 (Esophageal varices). However, the guidelines for coding hemorrhage state that if the hemorrhage is the principal diagnosis, it should be coded first. The specific code for esophageal varices with hemorrhage is I85.01 (Esophageal varices with hemorrhage). This code directly captures both the underlying condition (esophageal varices) and the acute event (hemorrhage). The physician’s documentation clearly links the hemorrhage to the esophageal varices. Therefore, I85.01 is the most appropriate principal diagnosis code. The documentation also mentions that the patient has a history of alcoholic cirrhosis, which is a common etiology for esophageal varices. While cirrhosis is a significant underlying condition, the immediate reason for the encounter is the bleeding. ICD-10-CM guidelines generally direct coders to code the condition that occasions the admission. In this instance, the hemorrhage is the acute event driving the medical necessity for the encounter. If the cirrhosis were the primary focus of treatment during this specific encounter, or if it were a more severe manifestation requiring specific management beyond the bleeding itself, it might be coded differently or as a secondary diagnosis. However, based on the provided documentation, the hemorrhage from the varices is the principal concern. Therefore, the correct coding sequence begins with I85.01.
-
Question 26 of 30
26. Question
A patient admitted to the hospital presents with a history of significant alcohol abuse and is found to have vomiting of blood (hematemesis) and dark, tarry stools (melena). The physician’s diagnostic assessment confirms the presence of esophageal varices as the cause of the bleeding. Considering the principles of accurate medical coding and billing as emphasized in the CMCB University program, which combination of ICD-10-CM codes best captures the patient’s clinical presentation and diagnosis?
Correct
The scenario describes a patient presenting with symptoms indicative of a gastrointestinal bleed. The physician documents “melena” and “hematemesis,” which are key terms. Melena refers to dark, tarry stools, typically caused by digested blood from the upper gastrointestinal tract. Hematemesis is the vomiting of blood. The physician also notes a diagnosis of “esophageal varices,” which are swollen veins in the esophagus, often associated with liver disease, and a common cause of severe gastrointestinal bleeding. To accurately code this encounter for CMCB University’s curriculum, we must select the ICD-10-CM codes that best represent the patient’s conditions and the physician’s documentation. The primary diagnosis is the esophageal varices, as this is the underlying condition leading to the bleeding. The bleeding itself is a manifestation of the varices. The ICD-10-CM code for esophageal varices is K76.6 (Esophageal varices). This code specifically addresses the presence of varices in the esophagus. The symptoms of melena and hematemesis, while important clinically, are considered manifestations of the underlying condition (esophageal varices) in this context. ICD-10-CM guidelines generally direct coders to code the underlying condition when it is known and the cause of the symptoms. While there are codes for melena (K92.1) and hematemesis (K92.0), these would typically be used if the underlying cause was not documented or if the bleeding was the primary focus without a specific etiology identified. However, given the explicit documentation of esophageal varices as the cause, K76.6 is the principal diagnosis. Therefore, the most appropriate coding approach, aligning with the principles taught at CMCB University for accurate and specific documentation, is to assign the code for the underlying pathology.
Incorrect
The scenario describes a patient presenting with symptoms indicative of a gastrointestinal bleed. The physician documents “melena” and “hematemesis,” which are key terms. Melena refers to dark, tarry stools, typically caused by digested blood from the upper gastrointestinal tract. Hematemesis is the vomiting of blood. The physician also notes a diagnosis of “esophageal varices,” which are swollen veins in the esophagus, often associated with liver disease, and a common cause of severe gastrointestinal bleeding. To accurately code this encounter for CMCB University’s curriculum, we must select the ICD-10-CM codes that best represent the patient’s conditions and the physician’s documentation. The primary diagnosis is the esophageal varices, as this is the underlying condition leading to the bleeding. The bleeding itself is a manifestation of the varices. The ICD-10-CM code for esophageal varices is K76.6 (Esophageal varices). This code specifically addresses the presence of varices in the esophagus. The symptoms of melena and hematemesis, while important clinically, are considered manifestations of the underlying condition (esophageal varices) in this context. ICD-10-CM guidelines generally direct coders to code the underlying condition when it is known and the cause of the symptoms. While there are codes for melena (K92.1) and hematemesis (K92.0), these would typically be used if the underlying cause was not documented or if the bleeding was the primary focus without a specific etiology identified. However, given the explicit documentation of esophageal varices as the cause, K76.6 is the principal diagnosis. Therefore, the most appropriate coding approach, aligning with the principles taught at CMCB University for accurate and specific documentation, is to assign the code for the underlying pathology.
-
Question 27 of 30
27. Question
A patient presented to CMCB University Medical Center for a routine screening colonoscopy. The physician documented the procedure as a screening colonoscopy with removal of a single sessile polyp via snare cautery. The pathology report subsequently confirmed the polyp to be a tubular adenoma. Considering the documentation and pathology findings, which ICD-10-CM code best represents the principal diagnosis for this encounter at CMCB University Medical Center?
Correct
The scenario describes a patient undergoing a diagnostic colonoscopy with polyp removal. The primary diagnosis is a screening for polyps, indicated by Z12.11 (Encounter for screening for malignant neoplasm of colon). During the procedure, a single sessile polyp is identified and removed via snare cautery. The pathology report confirms the polyp is a tubular adenoma, which is a precancerous condition. According to ICD-10-CM Official Guidelines for Coding and Reporting, when a screening colonoscopy leads to the discovery and removal of a lesion or polyp, the reason for the encounter changes from screening to diagnostic. Therefore, the primary diagnosis code should reflect the finding and management of the polyp. The appropriate ICD-10-CM code for a tubular adenoma found during a colonoscopy is K63.5 (Polyp of colon). The colonoscopy procedure itself is coded using CPT. The removal of a single polyp using snare cautery during a colonoscopy is reported with CPT code 45385 (Colonoscopy with removal of polyp(s), any method, including snare, …, by snare technique). The screening diagnosis (Z12.11) is not the principal diagnosis when a condition is found and treated during the screening. The presence of the tubular adenoma (K63.5) becomes the principal diagnosis because it was actively managed. The question asks for the most appropriate principal diagnosis code for the encounter.
Incorrect
The scenario describes a patient undergoing a diagnostic colonoscopy with polyp removal. The primary diagnosis is a screening for polyps, indicated by Z12.11 (Encounter for screening for malignant neoplasm of colon). During the procedure, a single sessile polyp is identified and removed via snare cautery. The pathology report confirms the polyp is a tubular adenoma, which is a precancerous condition. According to ICD-10-CM Official Guidelines for Coding and Reporting, when a screening colonoscopy leads to the discovery and removal of a lesion or polyp, the reason for the encounter changes from screening to diagnostic. Therefore, the primary diagnosis code should reflect the finding and management of the polyp. The appropriate ICD-10-CM code for a tubular adenoma found during a colonoscopy is K63.5 (Polyp of colon). The colonoscopy procedure itself is coded using CPT. The removal of a single polyp using snare cautery during a colonoscopy is reported with CPT code 45385 (Colonoscopy with removal of polyp(s), any method, including snare, …, by snare technique). The screening diagnosis (Z12.11) is not the principal diagnosis when a condition is found and treated during the screening. The presence of the tubular adenoma (K63.5) becomes the principal diagnosis because it was actively managed. The question asks for the most appropriate principal diagnosis code for the encounter.
-
Question 28 of 30
28. Question
A patient is seen at a CMCB University affiliated clinic with complaints of dysuria and increased urinary frequency. The physician’s progress note states, “Diagnosis: Cystitis.” Which ICD-10-CM code accurately reflects this diagnosis for billing purposes, assuming no further details regarding the type of cystitis or presence of hematuria are documented?
Correct
The scenario describes a patient presenting with symptoms indicative of a urinary tract infection (UTI). The physician documents the diagnosis of “cystitis.” To accurately code this condition for billing purposes at CMCB University, the coder must consult the ICD-10-CM coding manual. Cystitis, inflammation of the bladder, is classified under diseases of the genitourinary system. Specifically, ICD-10-CM code N30.90 represents “Cystitis, unspecified” without hematuria. The presence of hematuria would necessitate a different code (N30.00 for acute cystitis with hematuria, or N30.91 for unspecified cystitis with hematuria). Since the documentation only mentions cystitis and does not specify acute or chronic, nor does it mention hematuria, the most appropriate and specific code is N30.90. This aligns with the principle of coding to the highest level of specificity supported by the documentation, a core tenet of accurate medical coding taught at CMCB University. Understanding the hierarchical structure of ICD-10-CM and the nuances of diagnostic specificity is crucial for proper reimbursement and compliance.
Incorrect
The scenario describes a patient presenting with symptoms indicative of a urinary tract infection (UTI). The physician documents the diagnosis of “cystitis.” To accurately code this condition for billing purposes at CMCB University, the coder must consult the ICD-10-CM coding manual. Cystitis, inflammation of the bladder, is classified under diseases of the genitourinary system. Specifically, ICD-10-CM code N30.90 represents “Cystitis, unspecified” without hematuria. The presence of hematuria would necessitate a different code (N30.00 for acute cystitis with hematuria, or N30.91 for unspecified cystitis with hematuria). Since the documentation only mentions cystitis and does not specify acute or chronic, nor does it mention hematuria, the most appropriate and specific code is N30.90. This aligns with the principle of coding to the highest level of specificity supported by the documentation, a core tenet of accurate medical coding taught at CMCB University. Understanding the hierarchical structure of ICD-10-CM and the nuances of diagnostic specificity is crucial for proper reimbursement and compliance.
-
Question 29 of 30
29. Question
During a patient encounter at CMCB University Medical Center, the attending physician’s progress note details the patient’s presentation with significant hematemesis and the subsequent observation of melena in their stool. The physician’s assessment indicates a suspected upper gastrointestinal bleed, but a definitive diagnosis of the underlying cause, such as a peptic ulcer or gastritis, has not yet been confirmed. Considering the principles of accurate medical coding as taught at CMCB University, which ICD-10-CM code most precisely reflects the documented clinical findings in the absence of a confirmed etiology?
Correct
The scenario describes a patient presenting with symptoms suggestive of a gastrointestinal bleed. The physician documents “hematemesis” and “melena.” Hematemesis refers to the vomiting of blood, and melena refers to the passage of dark, tarry stools, indicating digested blood. In ICD-10-CM, the primary diagnosis for hematemesis is R19.03 (Vomiting of blood). For melena, the appropriate code is K92.1 (Melena). When multiple conditions are present, the coder must determine the principal diagnosis and any secondary diagnoses. In this case, both hematemesis and melena are significant findings related to the gastrointestinal bleeding. However, the question asks for the *most specific* code that captures the underlying pathology indicated by the documentation. While R19.03 accurately describes the symptom of vomiting blood, and K92.1 describes the symptom of melena, neither directly captures the *cause* of the bleeding, which is often a more critical piece of information for reimbursement and clinical management. The documentation implies a bleeding source within the gastrointestinal tract. Without further information specifying the exact location or cause (e.g., ulcer, diverticulitis), the coder must select the most encompassing code for the observed bleeding. Considering the options provided, K92.89 (Other specified diseases of the digestive system) is a less specific code that could encompass various unspecified digestive issues. K29.00 (Gastritis, unspecified, without bleeding) is incorrect as it explicitly states “without bleeding.” K29.01 (Gastritis, unspecified, with bleeding) is a more appropriate code if gastritis is the confirmed or suspected cause of the bleeding, as it directly links the inflammation to the bleeding event. However, the documentation provided is limited to the *symptoms* of bleeding (hematemesis and melena), not a confirmed diagnosis of gastritis. Therefore, the most accurate coding approach, based *solely* on the provided symptoms of vomiting blood and melena, would be to code the symptoms themselves if a definitive diagnosis is not yet established. ICD-10-CM guidelines often direct coders to code signs and symptoms when a definitive diagnosis has not been established. In this context, R19.03 for hematemesis and K92.1 for melena are the most direct symptom codes. However, the question asks for a single code that best represents the situation. When both hematemesis and melena are present, they are manifestations of gastrointestinal bleeding. ICD-10-CM provides a code for “Melena” (K92.1) which is a significant indicator of upper GI bleeding. Hematemesis is also a direct symptom. If the physician’s documentation strongly suggests an upper GI bleed without specifying the exact etiology (like an ulcer or gastritis), and melena is present, K92.1 is a strong contender. However, the question implies a need to capture the bleeding itself. Let’s re-evaluate the options in light of capturing the bleeding event. K92.1 (Melena) is a symptom of bleeding. R19.03 (Vomiting of blood) is another symptom. K29.01 (Gastritis, unspecified, with bleeding) assumes gastritis as the cause. K92.89 (Other specified diseases of the digestive system) is too general. The most accurate representation of the *clinical presentation* of significant gastrointestinal bleeding, as evidenced by both hematemesis and melena, without a confirmed underlying diagnosis, would be to code the most significant or representative symptom if a single code is required. Melena is often indicative of a more substantial bleed, particularly from the upper GI tract. Therefore, K92.1 is a strong candidate. However, the prompt is asking for the *most specific* code. If the physician documented both hematemesis and melena, and the underlying cause is not yet determined, coding the most significant symptom is often the practice. In many clinical scenarios, melena is considered a more definitive indicator of significant GI bleeding than hematemesis alone, especially when upper GI bleeding is suspected. Therefore, K92.1 is the most appropriate single code to represent the observed bleeding phenomenon when a specific etiology is not yet established. The calculation is not a mathematical one but a selection based on clinical documentation and ICD-10-CM coding guidelines. The process involves: 1. Identifying the documented symptoms: hematemesis and melena. 2. Understanding the meaning of these terms: vomiting of blood and dark, tarry stools, respectively. 3. Consulting ICD-10-CM to find the most specific codes for these symptoms. R19.03 for hematemesis and K92.1 for melena. 4. Recognizing that the question asks for the *most specific* code that captures the essence of the clinical presentation when a definitive underlying diagnosis is not yet established. 5. Evaluating the provided options against the documented symptoms and coding principles. 6. Determining that melena (K92.1) is a highly specific indicator of gastrointestinal bleeding, often from the upper GI tract, and when presented alongside hematemesis, it represents a significant clinical event that requires accurate coding. The other options are either incorrect (K29.00) or less specific (K92.89) or assume a diagnosis not explicitly stated (K29.01). Therefore, K92.1 is the most appropriate choice to represent the observed bleeding phenomenon in this scenario.
Incorrect
The scenario describes a patient presenting with symptoms suggestive of a gastrointestinal bleed. The physician documents “hematemesis” and “melena.” Hematemesis refers to the vomiting of blood, and melena refers to the passage of dark, tarry stools, indicating digested blood. In ICD-10-CM, the primary diagnosis for hematemesis is R19.03 (Vomiting of blood). For melena, the appropriate code is K92.1 (Melena). When multiple conditions are present, the coder must determine the principal diagnosis and any secondary diagnoses. In this case, both hematemesis and melena are significant findings related to the gastrointestinal bleeding. However, the question asks for the *most specific* code that captures the underlying pathology indicated by the documentation. While R19.03 accurately describes the symptom of vomiting blood, and K92.1 describes the symptom of melena, neither directly captures the *cause* of the bleeding, which is often a more critical piece of information for reimbursement and clinical management. The documentation implies a bleeding source within the gastrointestinal tract. Without further information specifying the exact location or cause (e.g., ulcer, diverticulitis), the coder must select the most encompassing code for the observed bleeding. Considering the options provided, K92.89 (Other specified diseases of the digestive system) is a less specific code that could encompass various unspecified digestive issues. K29.00 (Gastritis, unspecified, without bleeding) is incorrect as it explicitly states “without bleeding.” K29.01 (Gastritis, unspecified, with bleeding) is a more appropriate code if gastritis is the confirmed or suspected cause of the bleeding, as it directly links the inflammation to the bleeding event. However, the documentation provided is limited to the *symptoms* of bleeding (hematemesis and melena), not a confirmed diagnosis of gastritis. Therefore, the most accurate coding approach, based *solely* on the provided symptoms of vomiting blood and melena, would be to code the symptoms themselves if a definitive diagnosis is not yet established. ICD-10-CM guidelines often direct coders to code signs and symptoms when a definitive diagnosis has not been established. In this context, R19.03 for hematemesis and K92.1 for melena are the most direct symptom codes. However, the question asks for a single code that best represents the situation. When both hematemesis and melena are present, they are manifestations of gastrointestinal bleeding. ICD-10-CM provides a code for “Melena” (K92.1) which is a significant indicator of upper GI bleeding. Hematemesis is also a direct symptom. If the physician’s documentation strongly suggests an upper GI bleed without specifying the exact etiology (like an ulcer or gastritis), and melena is present, K92.1 is a strong contender. However, the question implies a need to capture the bleeding itself. Let’s re-evaluate the options in light of capturing the bleeding event. K92.1 (Melena) is a symptom of bleeding. R19.03 (Vomiting of blood) is another symptom. K29.01 (Gastritis, unspecified, with bleeding) assumes gastritis as the cause. K92.89 (Other specified diseases of the digestive system) is too general. The most accurate representation of the *clinical presentation* of significant gastrointestinal bleeding, as evidenced by both hematemesis and melena, without a confirmed underlying diagnosis, would be to code the most significant or representative symptom if a single code is required. Melena is often indicative of a more substantial bleed, particularly from the upper GI tract. Therefore, K92.1 is a strong candidate. However, the prompt is asking for the *most specific* code. If the physician documented both hematemesis and melena, and the underlying cause is not yet determined, coding the most significant symptom is often the practice. In many clinical scenarios, melena is considered a more definitive indicator of significant GI bleeding than hematemesis alone, especially when upper GI bleeding is suspected. Therefore, K92.1 is the most appropriate single code to represent the observed bleeding phenomenon when a specific etiology is not yet established. The calculation is not a mathematical one but a selection based on clinical documentation and ICD-10-CM coding guidelines. The process involves: 1. Identifying the documented symptoms: hematemesis and melena. 2. Understanding the meaning of these terms: vomiting of blood and dark, tarry stools, respectively. 3. Consulting ICD-10-CM to find the most specific codes for these symptoms. R19.03 for hematemesis and K92.1 for melena. 4. Recognizing that the question asks for the *most specific* code that captures the essence of the clinical presentation when a definitive underlying diagnosis is not yet established. 5. Evaluating the provided options against the documented symptoms and coding principles. 6. Determining that melena (K92.1) is a highly specific indicator of gastrointestinal bleeding, often from the upper GI tract, and when presented alongside hematemesis, it represents a significant clinical event that requires accurate coding. The other options are either incorrect (K29.00) or less specific (K92.89) or assume a diagnosis not explicitly stated (K29.01). Therefore, K92.1 is the most appropriate choice to represent the observed bleeding phenomenon in this scenario.
-
Question 30 of 30
30. Question
A patient presents to the clinic with a complaint of bright red blood in their stool. The physician documents “hematochezia” in the progress note and orders an esophagogastroduodenoscopy (EGD) to investigate the source of bleeding. Considering the principles of medical coding and the information provided in the initial documentation at CMCB University’s affiliated teaching hospital, what is the most appropriate initial coding action for the documented symptom?
Correct
The scenario describes a patient presenting with symptoms indicative of a gastrointestinal bleed. The physician documents “hematochezia” and orders an esophagogastroduodenoscopy (EGD). Hematochezia refers to the passage of fresh, red blood per rectum, typically originating from the lower gastrointestinal tract. However, the physician’s subsequent diagnostic procedure, an EGD, is specifically designed to visualize the esophagus, stomach, and duodenum – the upper gastrointestinal tract. This discrepancy highlights a potential disconnect between the initial symptom description and the diagnostic approach. In medical coding, accurate representation of both the patient’s condition and the services rendered is paramount. When a symptom is documented, and a more definitive diagnosis is established through diagnostic procedures, the coder must prioritize the diagnosis code if it is supported by the physician’s documentation and the diagnostic findings. However, the question specifically asks about the *initial* coding consideration based on the *provided documentation*. The term “hematochezia” itself is a symptom. While it points towards bleeding, it doesn’t specify the location or cause. The EGD, while ordered, is a procedure to *investigate* the cause. Therefore, the most appropriate initial coding approach, based solely on the documented symptom and the physician’s intent to investigate, is to code the symptom itself. ICD-10-CM provides specific codes for symptoms when a definitive diagnosis has not yet been established. In this case, R19.39, “Other and unspecified abdominal pain,” is not directly related to hematochezia. A more appropriate symptom code for hematochezia would be R19.11, “Hematochezia.” However, given the options, we must select the best fit. The physician’s documentation of “hematochezia” directly describes the observed symptom. The EGD is a diagnostic procedure to identify the source of the bleeding. Without the results of the EGD, the definitive diagnosis is not yet established. Therefore, coding the symptom is the most accurate initial step according to coding guidelines when a definitive diagnosis is pending. The physician’s choice of EGD suggests a suspicion of upper GI bleeding, but the symptom itself is what is documented as present. Coding the symptom ensures that the patient’s presentation is captured, even if the underlying cause is not yet confirmed. This aligns with the principle of coding what is known and documented.
Incorrect
The scenario describes a patient presenting with symptoms indicative of a gastrointestinal bleed. The physician documents “hematochezia” and orders an esophagogastroduodenoscopy (EGD). Hematochezia refers to the passage of fresh, red blood per rectum, typically originating from the lower gastrointestinal tract. However, the physician’s subsequent diagnostic procedure, an EGD, is specifically designed to visualize the esophagus, stomach, and duodenum – the upper gastrointestinal tract. This discrepancy highlights a potential disconnect between the initial symptom description and the diagnostic approach. In medical coding, accurate representation of both the patient’s condition and the services rendered is paramount. When a symptom is documented, and a more definitive diagnosis is established through diagnostic procedures, the coder must prioritize the diagnosis code if it is supported by the physician’s documentation and the diagnostic findings. However, the question specifically asks about the *initial* coding consideration based on the *provided documentation*. The term “hematochezia” itself is a symptom. While it points towards bleeding, it doesn’t specify the location or cause. The EGD, while ordered, is a procedure to *investigate* the cause. Therefore, the most appropriate initial coding approach, based solely on the documented symptom and the physician’s intent to investigate, is to code the symptom itself. ICD-10-CM provides specific codes for symptoms when a definitive diagnosis has not yet been established. In this case, R19.39, “Other and unspecified abdominal pain,” is not directly related to hematochezia. A more appropriate symptom code for hematochezia would be R19.11, “Hematochezia.” However, given the options, we must select the best fit. The physician’s documentation of “hematochezia” directly describes the observed symptom. The EGD is a diagnostic procedure to identify the source of the bleeding. Without the results of the EGD, the definitive diagnosis is not yet established. Therefore, coding the symptom is the most accurate initial step according to coding guidelines when a definitive diagnosis is pending. The physician’s choice of EGD suggests a suspicion of upper GI bleeding, but the symptom itself is what is documented as present. Coding the symptom ensures that the patient’s presentation is captured, even if the underlying cause is not yet confirmed. This aligns with the principle of coding what is known and documented.